Anda di halaman 1dari 431
Solutions to LE. Irodov’s Problems in General Physics Volume I Mechanics e Heat e Electrodynamics SECOND EDITION ABHAY KUMAR SINGH Director Abhay's I.1,T. Physics Teaching Centre Patni a \ cBSsS CBS PUBLISHERS & DISTRIBUTORS 4596/1A, 11 DARYAGANMJ, NEW DELHI - 110 002 (INDIA) Dedicated to my Teacher Prof. (Dr.) J. Thakur (Department of Physics, Patna University, Patna-4) ISBN : 81-239-0399-5 First Edition : 1995 Reprint : 1997 Second Edition : 1998 Reprint : 2000 Reprint : 2001 Reprint : 2002 Reprint : 2003 Reprint : 2004 Reprint : 2005 Copyright © Author & Publisher Alll rights reserved. No part of this book may, be reproduced or transmitted in any form or by any means, electronic or mechanical, including photocopying, recording, or any information storage and retrieval system without permission, in writing, from the publisher. Published by S.K. Jain for CBS Publishers & Distributors, 4596/1A, 11 Darya Ganj, New Delhi - 110 002 (India) Printed at : India Binding House, Delhi - 110 032 FOREWORD Science, in general, and physics, in particular, have evolved out of man’s quest to know beyond unknowns. Matter, radiation and their mutual interactions are basically studied in physics. Essentially, this is an experimental science. By observing appropriate phenomena in nature one arrives at a set of rules which goes to establish some basic fundamental concepts. Entire physics rests on them. Mere knowledge of them is however not enough. Ability to apply them to real day-to-day problems is required. Prof. Irodov’s book contains one such set of numerical exercises spread over a wide spectrum of physical disciplines. Some of the problems of the book long appeared to be notorious to pose serious challenges to students as well as to their teachers. This book by Prof. Singh on the solutions of problems of Irodov’s book, at the outset, seems to remove the sense of awe which at one time prevailed. Traditionally a difficult exercise to solve continues to draw the attention of concerned persons over a sufficiently long time. Once a logical solution for it becomes available, the difficulties associated with its solutions are forgotten very soon. This statement is not only valid for the solutions of simple physical problems but also to various physical phenomena. Nevertheless, Prof. Singh’s attempt to write a book of this magnitude deserves an all out praise. His ways of solving problems are elegant, straight forward, simple and direct. By writing this book he has definitely contributed to the cause of physics education. A word of advice to its users is however necessary. The solution to a particular problem as given in this book is never to be consulted unless an all out effort in solving it independently has been already made. Only by such judicious uses of this book one would be able to reap better benefits out of it. As a teacher who has taught physics and who has been in touch with physics curricula at 11.T., Delhi for over thirty years, I earnestly feel that this book will certainly be of benefit to younger students in their formative years. Dr. Dilip Kumar Roy Professor of Physics Indian Institute of Technology, Delhi New Delhi-110016. FOREWORD A. proper understanding of the physical laws and principles that govern nature require solutions of related problems which exemplify the principle in question and leads to a better grasp of the principles involved. It is only through experiments or through solutions of multifarious problem-oriented questions can a student master the intricacies and fall outs of a physical law. According to Ira M. Freeman, professor of physics of the state university of new Jersy at Rutgers and author of ‘‘physic--principles and Insights”” -- “In certain situations mathematical formulation actually promotes intuitive understand- ing, Sometimes a mathematical formulation is not feasible, so that ordinary language must take the place of mathematics in both roles. However, Mathematics is far more rigorous and its concepts more precise than those of language. Any science that is able to make extensive use of mathematical symbolism and procedures is justly called an exact science’’. I.E, Irodov’s problems in General Physics fulfills such a need. This book originally published in Russia contains about 1900 problems on mechanics, thermody- namics, molecular physics, electrodynamics, waves and oscillations, optics, atomic and nuclear physics. The book has survived the test of class room for many years as is evident from its number of reprint editions, which have appeared since the first English edition of 1981, including an Indian Edition at affordable price for Indian students. Abhay Kumar Singh’s present book containing solutions to Dr. I.E. Irodov’s Problems in General Physics is a welcome attempt to develop a student’s problem solving skills. The book should be very useful for the students studying a general course in physics and also in developing their skills to answer questions normally encountered in national level entrance examinations conducted each year by various bodies for admissions to profes- sional colleges in science and technology. BP. PAL Professor of Physics LLT., Delhi PREFACE TO THE SECOND EDITION Nothing succeeds like success, they say. Now, consequent upon the warm welcome on the part of students and the teaching fraternity this revised and enlarged edition of this volume is before you. In order to make it more up-to-date and viable, a large number of problems have been streamlined with special focus on the complicated and ticklish ones, to cater to the needs of the aspiring students. I extend my deep sense of gratitude to all those who have directly or indirectly engineered the cause of its existing status in the book world. Patna June 1997 Abhay Kumar Singh PREFACE TO THE FIRST EDITION When you invisage to write a book of solutions to problems, one pertinent question crops up in the mind that—why solution! Is this to prove one’s erudition? My only defence against this is that the solution is a challenge to save the scientific man hours by channelizing thoughts in a right direction. The book entitled “Problems in General Physics” authored by LE. Irodov (a noted Russian physicist and mathematician) contains 1877 intriguing problems divided into six chapters. After the acceptance of my first book “Problems in Physics”, published by Wiley Eastern Limited, I have got the courage to acknowledge the fact that good and honest ultimately win in the market place. This stimulation provided me insight to come up with my second attempt—‘Solutions to I.E. Irodov’s Problems in General Physics.” This first volume encompasses solutions of first three chapters containing 1052 problems. Although a large number of problems can be solved by different methods, I have adopted standard methods and in many of the problems with helping hints for other methods. In the solutions of chapter three, the emf of a cell is represented by & (xi) in contrast to the notation used in figures and in the problem book, due to some printing difficulty. I am thankful to my students Mr. Omprakash, Miss Neera and Miss Punam for their valuable co-operation even in my hard days while authoring the present book. I am also thankful to my younger sister Prof. Ranju Singh, my younger brother Mr. Ratan Kumar Singh, my junior friend Miss Anupama Bharti, other well wishers and friends for their emotional Support. At last and above all I am grateful to my Ma and Pappaji for their blessings and encouragement. ABHAY KUMAR SINGH CONTENTS Foreword Preface to the second edition Preface to the first edition 21 23 24 25 2.6 27 PART ONE PHYSICAL FUNDAMENTALS OF MECHANICS Kinematics The Fundamental Equation of Dynamics Laws of Conservation of Energy, Momemtum, and Angular Momentum Universal Gravitation Dynamics of a Solid Body Elastic Deformations of a Solid Body Hydrodynamics Relativistic Mechanics PART TWO THERMODYNAMICS AND MOLECULAR PHYSICS Equation of the Gas State. Processes The first Law of Thermodynamics. Heat Capacity Kinetic theory of Gases. Boltzmann’s Law and Maxwell’s Distribution The Second Law of Thermodynamics. Entropy Liquids. Capillary Effects Phase Transformations Transport Phenomena PART THREE ELECTRODYNAMICS Constant Electric Field in Vacuum Conductors and Dielectrics in an Electric Field Electric Capacitance. Energy of an Electric Field Electric Current Constant Magnetic Field. Magnetics Electromagnetic Induction. Maxwell’s Equations Motion of Charged Particles in Electric and Magnetic Fields iii vi 1-34 35-65 66-101 102-117 118-143 144-155 156-167 168-183 184-195 196-212 213-226 227-241 242-247 248-256 257-266 267-288 289-305 306-324 325-353 354-379 380-407 408-424 PART ONE PHYSICAL FUNDAMENTALS OF MECHANICS 11 KINEMATICS 1.1 Let v, be the stream velocity and v’ the velocity of motorboat with respect to water. The motorboat reached point B while going downstream with velocity (v, + v’) and thea returned with velocity (v' - vy) and passed the raft at point C. Let ¢ be the time for the raft (which flows with stream with velocity v,) to move from point A to C, during which the motorboat moves from A to B and then from B to C. Therefore oe Guest — Cae ed) A 8 S On solving we get v, = + | 12 Let s be the total distance traversed by the point and #, the time taken to cover half the distance. Further let 2¢ be the time to cover the rest half of the distance. s Therefore x7 OF he Be (1) s s and Zo ty)t or Um 2) Hence the sought average velocity : 2¥4 (v4 +¥%) s Hence At=etV 1- = 15s. wt (a) Sought average velocity ee eee t” 20s (®) For the maximum velocity, s should be maximum. From the figure & is maximum for all points on the line ‘ac, thus the sought maximum velocity becomes average velocity for the line ac and is equal to : be | 100.cm a ae 3 = 25 cm/s (©) Time fy should be such that corresponding to it the slope é should pass through the point O (origin), to satisfy the relationship 4. = From figure the tangent at point d 0 passes through the origin and thus corresponding time f= fy = 165. 1.5 Let the particles collide at the point A (Fig,), whose position vector is 73 (say). If ¢ be the 16 time taken by each particle to reach at point A, from triangle law of vector addition : en by ach particle t Ta= 7, +¥ t= ro t+vyt Sle ate A so, Hae @G-a)e ® @F ve therefore, .- Hara (2) ae rd : Iv2-%] % Vt From Eqs. (1) and (2) ee ia = ee ier) 0 ® Ree Bea * or, tS = 3S, which is the sought relationship. In-rl lve’ We shave a 2 @ From the vector diagram [of Eq. (1)] and using properties of triangle 17 va Vite? 42vgvcosp = 39.7km/br (2) v and = or, sin = EP sin(n-g) sin® yv or O= sin” Using (2) and putting the values of v and d O= 19.1" Let one of the swimmer (say 1) cross the river along AB, which is obviously the shortest path. Time taken to cross the river by the swimmer 1. d Vv28 For the other swimmer (say 2), which follows the quickest -path, the time taken to cross the river. 4 , (where AB = d is the width of the river) (6) Qe 4 Q) D Be—— x ——>6 In the time f,, drifting of the swimmer 2, becomes y X= V9l= yh (using Eq. 2) @) If f, be the time for swimmer 2 to walk the distance x to come from C toB (Fig.), then 2 %04 Gasing Ea, 3 4 f- <= 57, (using Ea. 3) (4) According to the problem f, = f+ 3 d a or, “7 v2aw On solving we get = 3km/hr. 4 18 19 Let I be the distance covered by the boat A along the river as well as by the boat B acrc the river. Let vp be the stream velocity and v’ the velocity of each boat with respect water. Therefore time taken by the boat A in its journey ue +s - ae v=% and for the boat B "Va z ~ve Hence, = ( where n = 5) On substitution ty/tg= 18 Let vp be the stream velocity and v’ the velocity of boat with respect to water. A Ve 3 1 = 2>0, some drifting of boat is inevitable. Let v” make an angle 6 with flow direction. (Fig.), then the time taken to cross the rive d t= 0 (where d is the width of the river) In this time intezval, the drifting of the boat x= (v' cos 8 + v9) t = (08 04) Se 2 G7 (cot +n cosec 8) — ‘OF Xin — drifting) — d ae 7 Zo (Cot 8 +m cosec 8) = 0, which yields y! . 1 1 vv, — cos0= -2.-2 0 n 2 x Hence, O= 120° 0 The solution of this problem becomes simple in the frame attached with one of the bodies. Let the body thrown straight up be 1 and the other body be 2, then for the body 1 in the frame of 2 from the kinematic equation for constant acceleration : os. lo 2 72 Foun * Yount + > M2! So, Fam Yount (because Wp = 0 and Fo12)= 0) or, l7gl= [¥oaa)lt q@) But |¥o,1= |¥al= % So, from properties of triangle Voxiny = V ve +8 - 2 v9 ¥9 c08 (2/2 ~ By) Hence, the sought distance |7yl= voV 2 -sin 8) t= 22m. 1 112 5 Let the velocities of the paricles (say vy and ¥]_) becomes mutually perpendicular after time ¢. Then their velocitis become me ee wet oe Vy = Vj +Bt; vy = v2 tee @ As vy Lip 80, vv) = 0 or, (+84): (+84) = 0 or -yy y+ t= 0 Hence, t= “_ 1 Now form the Eq. Fi = Taz + Youn! + zh ot l7yl= [Vaal (because here Wi, = 0 and 792) 9) Hence the sought distance mi = +2 => ale AEBV i Galtian l= v4 Yd) From the symmetry of the problem all the three points are always located at the vertices of equilateral triangles of varying side length and finally meet at the centriod of the initial equilateral triangle whose side length is a, in the sought time interval (say ¢). > = 120° 3° Let us consider an arbitrary equilateral triangle of edge length / (say). ‘Then the rate by which 1 approaches 2, 2 approches 3, and 3 approches 1, becomes : On integrating : 6 113 1.14 Let us locate the points A and B at an arbitrary instant of time (Fig.). If A and B are separated by the distance s at this moment, then the points converge or point A approaches B with velocity =s. v—ucos a where angle o. varies with time. On intergating, o ba ~fds= fv-ucos a) at, V o (where T is the sought time.) - or I= f(v-ucos.a) dt (Qy 0 As both A and B cover the same distance in x-direction during the sought time interval, so the other condition which is required, can be obtained by the equation Ax= fv.at r So, uT= fvcos a at 2) ° : ul Solving (1) and (2), we get T ant One can see that if u= v, or up and the time taken to travel the distance CB. A in the field Ce eee @ =\ x So, the total time elapsed to move the car from point A to B a) 1 Vee t= 4 + iy eee ee ees nv v For ¢ to be minimum \ 8 too o 1{-1,_4]-0 dc vpn VP ar or Wee Pa or x= 9 1.18 To plot x(¢),s(¢) and w, (f) let us partion the given plot v, (f) into five segments (for detailed analysis) as shown in the figure. For the part oa: w,= 1 and v.= t= v ' 2 Thus, Ax, (= fu,de= fae= oe 5,0 0 . Putting t= 1, we get, Ax, = s= $ unit For the part ab : w,= O and v= v= constant = 1 1 Thus An ()= fv,dt= fat = ¢-)= 5,0 1 Putting t= 3, Axy= s)= 2unit For the part b4: w,= 1 and v= 1-(t-3)=4-2)=¥ ' 2 ha 5) Thus oad ae 53 Putting t= 4, Ax=x3= S unit For the part 4d : v= -Land y= -(1-4)= 4-1 So, v= |v,[=1-4 for t>4 : 2 t Thus CSU ie 41-58 Putting t= 6, Ax,=-1 ' 2 oad ie Similarly Ga la) a eae Putting t= 6, s.= 2unit For the part d 7: w,= 2 and v,= -2+2(t-6)= 2(¢-7) v= |v, |= 2(7-2) for 1-7 ‘ Now, Ax (t)= [2 @-Tadt= 12-1404 48 1 Putting t= 4, Ars= -1 ‘ Similarly 53()= [2(7-) d= 1414-17-48 : Putting t= 7, soe On the basis of these obtained expressions w, (1), x(t) and s (f) plots can be easily plotted as shown in the figure of answershect. 10 1.19 (a) Mean velocity 1.20 Total distance covered Time elapsed oak vu eS SOcm/s = (1) 0 v (b) Modulus of mean velocity vector i f [|= = = A (eee Fig.) 6 Using (4) in (5), we get : : eo ews | ee ? (a) we have = at(l-ad) So, ve &, 7 -2a8) dt

oo 1.21 ll Hence, the sought distance 2a Va s= fvde= f a(1-2ande+ f a@at-War 0 ira orn a Simplifying, we get, s= a (a) As the particle leaves the origin at r= 0 So, Ace x= frat qa oof, tt As ve vol), where ¥ is directed towards the +ve x-axis So, v= Vy (: - 1} 2) From (1) and (2), vo fufi-ae voe(t-35] 8) 0 Hence x coordinate of the particle at r= 6s. 6 x= 10%6 {1-305} 24cm = 024m Similarly at and at 20 x 10% 20/1325] ~200em = -2m (b) At the moments the particle is at a distance of 10 cm from the origin, x = + 10 cm. Putting x= +10 in Eq. (3) 10= 10r(1--) or, t?-10r+10= 0, 10 So, to p= 12 VI00= 40. VIO 52 Vs Now putting x= ~10in Eqn (3) t -10= 10(1- io} On solving, t=5eVB 5 As t cannot be negative, so, t=(5+V35)s 12 1.22 Hence the particle is at a distance of 10 cm from the origin at three moments of time : t=S2Vi5 s,5+v35 s (c) We have ve voli -= We hi ve [1 -F £ vo(t-3) forts t So, ve |vy= v,(£-1)} fort>t ls So se vo(t-z)a for ts t = vgt(1- 4) «ote ' t t and on foft-s)arefult-s)a for t> 0 t = vt [1+ (1- 44/2 for t>e (A) 4 4 t t on fn(i-ga- frof-ga- 24cm, o 0 And for t= 8s 7 8 t t oo f0(t-Jae frogs} 0 5 On integrating and simplifying, we get s= 34cm. On the basis of Eqs. (3) and (4), x (¢) and s(t) plots can be drawn as shown in the answer sheet. As particle is in unidirectional motion it is directed along the x-axis all the time. As at t=0,x=0 av So, ar= xe sand Fm Therefore, veave= avs oe po oe Gras ae : dt 2vs dt 2vs av aavs_ oF “2ve7 ave” 2 a dv oe As, Vea 7 c ' o a? On integrating, fo-fea on, ve St (2) 1.23 1.24 13 (b) Let s be the time to cover first s m of the path. From the Eq. se fod ' 2 0) @ at : fs ae OF (using 2) 0 or t= 2 V5 @) The mean velocity of particle 2Vs/a ce fro | fa 2Vs/a 2 = According to the problem - wae = aVv (as v decreases with time) or, -f W dv= afas On integrating we get s= 2 Again according to the problem De ay or - = ade at we 0, dv or, Le afa Seed % Thus t= (a) As re atizbr 7” So, x= at, y=—-bi? 2 and therefore yer oe 14 1.25 which is Eq. of a parabola, whose graph is shown in the Fig. (b) As Fe atitbs?j” oe EL aitansr 1) ve a aie J (dl) So, v= Va2(-2bty = Vae+4ber Diff. Eq. (1) want. time, we get — dv v We oe -2bj So, [Wl =w=2b vw (ai=2btj)-(-2b6jF v0 Waa be) 2b 2bt or, ot= SF Va7+4br? 80, tana= =o : 2bt © cos a= or, a= ant { ae 2bt (d) The mean velocity vector ' fa [lai 2biFJae v = = ait? fa : Hence, ||= Va7 (bi)? = Va74b7 7 (a) We have x= atand y= at(l-at) (1) Hence, y (x) becomes, Ak (y_ OX), a2 ye (1 a) x- 2x? (parabola) (b) Ditferentiating Eq. (1) we get v= a and v= a(1-2a2) Q) 1.26 1.27 15 So, v= Wi 4 v7 =aV14(1-208)7 Diff. Eq. (2) with respect to time w,= 0 and w= -2a0 So, we Vwee wy = 2aa (6) From Eqs. (2) and (3) We have v= ajta(1-2ae)j” and We 2aaj” rw -a(1-2af,)2aa So, i e. + vw ( fo) 40 ¥2 vw aV1+(1-20%) 2a On simplifying. 1-20f= #1 1 As, tye 0, &= a Differentiating motion law : x= asinwt, y= a(1-coswr), with respect to time, ¥,= 4 COS Mf, v= awsin wt So, V= awcos at {+ asin tj () and v= a@= Const. Q) Differentiating Eq, (1) with respect to time = we e. -ae?sin oti a0? cos wt] 8) (a) The distance s traversed by the point during the time v is given by s=fvde= fawdt= aor (using?) 0 0 (b) Taking inner product of V” and Ww 2 =~ fe : = ‘We get, v? w= (a cos wtit+a sin wt; )- (ao sin wt (-i) +a o* cos wt -j) So, v7 We - a? w sin wt cos wt + a” wo sin wt cos wt = 0 si - x Thus, V°L W, i.c., the angle between velocity vector and acceleration vector equals >. Accordiing to the problem w= w(-j) dy, dy, a2 oto. So, w= Git 0 and w= = -w Q) Differentiating Eq. of trajectory, y= ax—- bx”, with respect to time d adx dx e- @) ee ae ~2bx 16 1.28 So, = jx-0 Again differentiating with respect to time dy_ad?x dx)" d?x ae tr 2(Z) 02Px a 2 dx : - ~w= (0)-20(F] ~2bx(0) (using 1) a : or, EA V ap (osing 1) ¢ “VE « Hence, the velocity of the particle at the origin 2 2 veV (a) (4)., = VE +0? (using Eqns (3) and (4)) Hence, ve VeU+e) Using (3) in (2) 2 x0 As the body is under gravity of constant accelration g’, it’s velocity vector and displacemen vectors are: ae vEy+et a and Are Fe Hee dee? (= Oate = 0) (em So, over the first ¢ seconds 2. oe Be A He @) Hence from Eq. (3), over the first t seconds <= ior be (4) For evaluating ¢, take VV (H+ +B) = vo +20 ere? or, = vet (voRlt+ et? But we have v= vp at ¢= 0 and Also at = + (Fig.) (also from energy conservation) 129 17 Hence using this propety in Eq. (5) ven va+2(VoRc+ er? 20-3 As t#0, so, t= -—>— g& Putting this value of t in Eq. (4), the average velocity over the time of flight ao loa = Fae ay 2) From Eqs. (1) and (2) sre, 5" | (480)? * (47? On simplifying 4 - 2400 1? + 1083750 = 0 20 1.33 1.34 Solving for +? we get : 2 2400 + V 1425000 2400 + 1194 2 2 Thus t= 4239s = 0-71 min and T= 2455s = 0-41 min depending on the angle a. Let the shells collide at the point P(x, y). If the first shell takes ¢ s to collide with second and At be the time interval between the firings, then X= vycos 0, t= vycos 0, (t-At) (1) and y= vpsin 01-5 gt? ¥ : 1 BT = vysin @,(¢~A)-Fe-ay @) Ly At cos 6, (oy) From Eq) t= Sg on By e |] P From Egs. (2) and (3) Ye 9 2 vp sin (8, - 0,) 0 ” Z(e0s0, +0080) * 4**° x According to the problem () Bam vy oF dy vyat : : eee f aon f dt or y= vot ay o o And also we have &. ay or de=aydt=avytdt (using 1) : i 2 So, fdr=avfid, o, x= davyi?= 2 (using 1) o oO (b) According to the problem vy= vp and v,= ay (2) So, oe R=Vviray dv yy dy Therefore Vujray dt Vy + (ay vo) Diff. Eq. (2) with respect to time. dv, dy, —2= w= 0 and —2= w= dt y dt a So, w= |wl= av 1.35 1.36 Vee Nee ee Hence w, ‘ Oe oi (win ae @ Tp (@) The velocity vector of the particle rd v= aitbxj So, &. a: 2. bx : ; From (1) fen ofa a o 0 And dy= bx dt= bat dt 7 ; Integrating fo- ab fede or y= dabs? 0 o From Eqs. (2) and (3), we get, y= ze (b) The curvature radius of trajectory y (x) is : ES 1+ (dy /dx)? i | Pysae | Let us differentiate the path Eq. y= ze with respect to x, ayo ays b aoa de a From Eqs. (5) and (6), the sought curvature radius : eal] In accordance with the problem wee But 0 2 or vam ds So, vdv= (@°t)ds= a-dr or, vdv= ai-dr= adx (because ais directed towards the x-axis) ’ : So, fra afar 0 0 Hence v= 2ax or, v= V2ax 21 qa) (2) @) @ 6) (6) 22 1.37 1.38 The velocity of the particle v= at av So, an Ww 2 And w= 5.8 ét (using v = at) (2) From se fvde ' 3 -2nRn= afvde Sat oO a aan? @ > a R From Egs. (2) and (3) w, = 4xan Hence w= V w+w? = Vas (4nany = aV1+160 Hy = 08 m/s? According to the problem Iw, t= [1 For v(t), 2. a Integrating this equation from vys v< v and Ox t= ¢ Se if ( ea 2 Now for v(s), - 1 %, Integrating this equation from vps vs v and Os ss 5 - s dv 1 v Ss SO, fe-tfe« Mt eer , ° 0 Hence vewe* (2) (b) The normal acceleration of the point 2 s/R w= Rt oe (using 2) And as accordance with the problem Iwl=|w,| and w,t,Lw, i, ve 2 s0, we VEw,= VE ge = VE 139 1.40 23 From the equation v= avs dads a & wn Be Be Sf avsn ©, and Zl @s ae Re oR As w, is a positive constant, the speed of the particle increases with time, and the tangential acceleration vector and velocity vector coincides in direction. Hence the angle between v°and w'is equal to between w, ig an W, and o can be found Iw,l_ a?s/R _ 2s by means of the formula : tana. =~ ” Iw" a2 R From the equation I= asinot ae Gace at So, w= 2a —atsinor, and (a) ¥ @ wr cos? wt wn be See ® (a) At the point J= 0,sinws= 0 and coswt= # 1 so, wt= 0, x etc. 2 Hence we woe Similarly at/= £ a, snot= + 1 and cos@f= 0, so, w,= 0 Hence we |w,|= ao? As w,= a and at = 0, the point is at rest So, v(@® and s(Q) are, v= at and s= par? (yy Let R be the curvature radius, then Ves aie w= oe "ROR But according to the ‘problem - 2s (using 1) w, = bt* a Zbs_ (sing 1) 2) Therefore w= Vw+w = Va?+(2as/R) = V @ + (4 bs? / a)” (using 2) Hence we aV 1+ (4087/0) 24 1.42 (a) Let us differentiate twice the path equation y (x) with respect to time. 2 2 Do nar Ht, Pe o4| (2) 4 Pe dt ear) at 2a at oe Since the particle moves uniformly, its acceleration at all points of the path is normal and at the point x = 0 it coincides with the direction of derivative d? y/dt?. Keeping in mind that at the point x = 0, Fie We get we #y =2av=w, a x= 0 y 1 So, wi 2av— BURG Note that we can also calculate it from the formula of problem (1.35 b) (b) Differentiating the equation of the trajectory with respect to time we see that Bx S ay 2. 0 (1) which implies that the vector (b’xi+ a*yjJ is normal to the velocity vector ve fr 2 J’ which, of course, is along the tangent. Thus the former vactor is along the normal and the normal component of acceleration is clearly pete, af ra (e+ aly onusing w,= wen/ |r]. Atx= 0, y= * band so atx =0 x0 Differentiating (1) 2 2 #(@) + a 3) “(¢) +a (#)- 0 Also from (1) ®oar=0 So (a) v (since tangential velocity is constant = v ) mm (4)-23° a wv wt yee 1.43 1.44 25 Let us fix the co-ordinate system at the point O as shown in the figure, such that the radius vector 7of point A makes an angle @ with x axis at the moment shown. Note that the radius vector of the particle A rotates clockwise and we here take line ox as reference line, so in this case obviously the angutar vetosiy o (42) aking anticlockwise sense of angular displacement as positive. Also from the geometry of the triangle OAC R r ind Tada) 1 7™ 2Revs 8. Let us write, 7= rcos 0 [4 rsin Oj= 2Rcos*j+ Rsin26j” Differentiating with respect to time, = or = 2R2cos 0(~sin 0) 227% 2. cos 2.022 7 our aR (=3° 2) {sin2 07° cos 207] or, = 2Rw (sin 20i—cos?0j) So, |v] or v= 20R=0-4m/s. As w is constant, v is also constant and w, = 2. 0, 2 vi vee Gory, 407R = 032 m/s? So, wewe Alternate : From the Fig. the angular velocity of the point A, with respect to centre of the circle C becomes 428) (-a0 dt dt Thus we have the problem of finding the velocity and acceleration of a particle moving along a circle of radius R with constant angular velocity 2 @. )-20- constant Hence v= 2oR and 2 (2wRP v wen Beet 4w?R Differentiating p(t) with respect to time 12 we 2at @) at For fixed axis rotation, the speed of the point A: v veoR=2atR or R= 5 (2) 26 1.45 1.46 Differentiating with respect to time w= #. 2aR~ ©, (using 1) 2 But Wee x ee (using 2) R_ w/2at So, wa Vwlew? =V(v/t)+(2atvy = iVie4a7r The shell acquires a constant angular acceleration at the same time as it accelerates linearly. The two are related by (assuming both are constant) wee 1 2nn Where w= linear acceleration and =_angular acceleration Then, w= V2B2a0= V 24 (2any But v?= 2wi, hence finally - 2unv 7 Let us take the rotation axis as z-axis whose positive direction is associated with the positive direction of the cordinate @, the rotation angle, in accordance with the right-hand screw rule (Fig.) (a) Defferentiating p(t) with respect to time. Z $80 g-3517= 0, (t) and ao do, oo = Gee B= -601 Q) From (1) the solid comes to stop at At= t= ve The angular velocity w= a-3bt?, for 0-= dt 0 Similarly B= |B,|= 6b for all values of £. 1.47 1.48 27 Va/; fear “Poors Sat Va “eps, 0 So, = = V3ab () From Eq. (2) B= - 6bt So, (B,),= Va73b = - 6b WE = -2Vab Hence B= | (82)... yarap | 7 24928 Angle a is related with |w,] and w, by means of the fomula : Wa Iw,]” where R is the radius of the circle which an arbitrary point of the body circumscribes. tana = where w,= @7R and |w,|= BR () From the given equation B = 42. at (here B= do » as B is positive for all values of 1) * ' Integrating within the limit [do = afrae or, on5ar? 2 4 and |w,|= BR» atR Putting the values of |w,| and w, in Eq. (1), we get, 244 3 at'R/4_ at 4 on tHls, at wy, 1 {(2) ana] 2\7 2,4 So, mo oR» (5) R= +R v3 In accordance with the problem, B, < 0 ‘Thus -2. k Vo, where k is proportionality constant do f kt or, “Live he or, Va = Vag - q@ When w = 0, total time of rotation t= t= 2 ‘ 28 1.49 1.50 2Van/k 1? f (one Spr -tevag ae a foar Average angular velocity < w > = = far 2Ve0 /k ee. peo! | 5 Hence < w >= | @yt+——~ > Veg You?) [2% - ays 12 k We have w= W)-a@ = _ Integratin this Eq. within its limit for (@) ¢ f Wp -k@ Ste iy ~ [as or, in *s =-kt Hence Qa Ba -e") @ (b) From the Eq., @ = wy k@ and Eq, (1) or by differentiating Eq. (1) o= oye" Lt us choose the positive direction of z-axis (stationary rotation axis) along the vector B,, In accordance with the equation do, do, i oe 6, or, 0,do,= B,dp= Boos pdg, Integrating this Eq. within its limit for w, () te e f4o.- of cos edo ° 0 oo or, Zz 7 Bosine Hence = V2 sing The plot «, (p) is shown in the Fig. It can be seen that as the angle @ grows, the vector @hirst increases, coinciding with the direction of the vector By (w, > 0), reaches the maximum at p= @/2, then starts decreasing and finally turns into zero at @ = x. After that the body starts rotating in the opposite direction in a similar fashion (w, < 0). As a result, the body will oscillate about the position p= p/2 with an amplitude equal to 1/2. 29 1.51 Rotating disc moves along the x-axis, in plane motion in x ~y plane. Plane motion of a solid can be imagined to be in pure rotation about a point (say /) at a certain instant known as instantaneous centre of rotation. The instantaneous axis whose positive sense is directed along @ of the solid and which passes through the point J, is known as instantaneous axis of rotation. Therefore the velocity vector of an arbitrary point (P) of the solid can be represented as : vps Ox Tp (1) On the basis of Eq. (1) for the C. M. (C) of the disc y v= Tx7y @ =a d? According to the problem v; tf i and Stt Fie. Gix-y plane, so to satisy the Eqn. Qe Toyis directed along (- j j). Hence point . xe Tis at a distance r¢,= y, above the centre of 0 the disc along y-axis. Using all these facts in Eq, (2), we get Ye Yom wy or y= @G) (a) From the angular kinematical equation O,= Oy +8,t @ o= Bt On the other hand x= vt, (where x is the x coordinate of the C.M.) x or, tat 6) From Eqs. (4) and (5), @ = Be Using this value of w in Eq. (3) we get pet = Seat ) ” Bx/v a. i x (b) As centre C moves with constant a w, with zero initial velocity So, x= Swe? and v,= wt Therefore, veo wy =vV2xw Ye V2wx Hence yo fo SX (parabola) 30 152 1.53 The plane motion of a solid can be imagined as the combination of translation of the C.M. and rotation about cM. a So, we may write y= Vo+Vc ee = tO Tae (1) and aS Wy WotWyo nWet07(~ncdt(Bkiac) 2 Tac is the position of vector of A with respect to Cs In the problem v,= v= constant, and the rolling is without slipping i.e, vo= v= oR, So, Wo = 0 and B= 0. Using these conditions in Eq. (2) > 2 2 A ve, a Wan ©? (~iAc)= OPR(~ike)= F(~te) Here, tiy¢ is the unit vector directed along rc 2 > A Hence w, = i and w, is directed along (-t,¢) or directed toward the centre of the wheel. (b) Let the centre of the wheel move toward right (positive x-axis) then for pure tolling on the rigid horizontal surface, wheel will have to rotate in clockwise sense. If w be the v angular velocity of the wheel then o = ze re Let the point A touches the horizontal surface at t= 0, further let us locate the point A att= ft, When it makes 6 = wf at the centre of the wheel. From Eqn. (1) Wy VerOX Tao = - - = vitw(-k)x [Ros 0 (-j) +R sin @(-i)] or, x vitwR[coswt(-i)+sinwtj } ad a = (¥-cos wf) i+ vsin wt] (as v= oR) So, v4 = V (v-vcos wt)? + (v sin wt)” = vV 2(1-cos wt) = 2vsin (wt /2) Hence distance covered by the point A during T= 2x/o 2x/w a= f ngdta f 2vsinoe/2) dew =. 8R. oO Let us fix the co-ordinate axis xyz as shown in the fig. As the ball rolls without slipping along the rigid surface so, on the basis of the solution of problem 1.52 : Mom Ve+OxT,= 0 Thus ce 7 v= oR and Ott (-k) as HITT @) 31 - @,+ Bx 73, = 0 and w= BR and Btt (-k) as wetti A VetWR=Va At the position corresponding to that of Fig., in accordance with the problem, B W.= W, SO V.= wt \ %¢ Me and o- sat and B= & (using 1) Us (a) Let us fix the co-ordinate system with the oe attached with the rigid surface as shown in the Fig. As point O is the instantaneous centre of rotation of the ball at the moment shown in Fig. so, v= 0, Now, Vg Vet Tae = Veit w(CE)XRG)* (e+ oR)E > Eee 7 So, v= 2vcim 2wei (using 1) | > ed Similalry Vp= 02+ Bx room voit o(-k)xR@ taal on 7 rd = vit OR (-J)* Veit Ve(-J) So, vg = V2 v,=VZ wt and Vp is at an angle 45° from both and j Fig.) (©) ym We+ 0 (7) + BR Fe o_o A W #&p = 0 (Fc) =F Coc) (using 1) where tigg is the unit vector along 7c \, 2 B $0, Wy= ee we (using 2) and iw is . directed towards the centre of the ball BR Now = We+0 Cie) + Baie a epee ee fee i = wita?R(-j)+B(-K)xRj = av os ae - wpa ees (using 1) = aie O75 A4 2\2 So, myo V awh te = 20 +(e 2R Similarly Wy = We + 0 (- 750) +B Fac = wis oR (-1)+ BCR) XRG) = [y- FE] FF BRT) (using 1) 32 . (w- T| i+w(-j (using 2) So, wa V ( T) + R 1.54 Let us draw the kinematical diagram of the rolling cylinder on the basis of the solutio: of problem 1.53. A Vaz2Vp Wet Br \ CO We Br 0 : 0 As, an arbitrary point of the cylinder follows a curve, its normal acceleration and radius of curvature are related by the well known equation w= R ' "A so, for point A, Wa (a) ™ R A 4 or, 4r (because v,= wr, for pure rolling) Similarly for point B, Wem) ~ Re v2 v,)? w*r cos 45° = ee 3 or, ¥ Rg= 2V2—<= 2VIr or 1.55 The angular velocity is a vector as infinitesimal rotation commute. Then the relative angular velocity of the body 1 with respect to the body 2 is clearly. > > > on ees as for relative linear velocity. The relative acceleration of 1 w.r.t. 2 is (@), 1.56 1.57 33 where S' is a frame corotating with the second body and S is a space fixed frame with origin coinciding with the point of intersection of the two axes, (S)-(F) a3 but = +2 xO; a ), dt); Since S’ rotates with angular velocity , . However ( 77 7 ‘| = 0 as the first body rotates s with constant angular velocity in space, thus Ba = 8 xi, Note that for any vector 5} the relation in space forced frame (k) and a frame (X) rotating with angular velocity @ is as" d — iB) =| tone K Wehave = atit br; (1) So, w= V (ay + (OF), thus, ol, 19, = 7.81 rad/s Differentiating Eq. (1) with respect to time Be 22 att 2b” Q) So, B= Vas (2b and Blea 10s = 1:3 rad/s? 0) cosa~ DE. (wit b?7) “(ait 2btj ) BV (an)? + (be) V a? + (bir Putting the values of (a) and (b) and’ taking t= 10s, we get aw 17° (a) Let the axis of the cone (OC) rotates in anticlockwise sense with constant angular velocity &’ and the cone itself about it’s own axis (OC) in clockwise sense with angular velocity @ (Fig.). Then the resultant angular velocity of the cone. Ba T +H, (1) As the rolling is pure the magnitudes of the vectors @ and @ can be easily found from Fig. oy ©" Reta” @y= V/R (2) As & 1G, from Eq. (1) and (2) 34 1.58 o= Vo7+o3 ve 2 v\2 Vv (a +] [kee (b) Vector of angular acceleration do d@ +H) B= ear 8 T= constant.) The vector @y which rotates about the OO’ axis with the angular velocity @, retains i magnitude. This increment in the time interval dt is equal to [d@y| = wg o' dt or in vector form dy = (@" x Gy) dt. Thus Be xd, G The magnitude of the vector Bris equal to B= ww (as D 155) voy B= Reota R So, - Sin a= 23 1ad/s The axis AB acquired the angular velocity w= Boe @ Using the facts of the solution of 1.57, the angular velocity of the body oz Vor+o” = Veg + B30? = 0-6 rad/s And the angular acceleration. Pe ao d@ +H) aa da dt dt dt dt day aw But “gi” @ XG, and = Boe So, Be (Botx Gy) + Bo As, ByLGy 0, Be V (0p By)” + Bo = By V1 + (pt) = 02 rad/s? 2 4.59 1.60 161 35 THE FUNDAMENTAL EQUATION OF DYNAMICS Let R be the constant upward thurst on the aerostat of mass m, coming down with a constant acceleration w. Applying Newton’s second law of motion for the aerostat in projection form F,= mw, mg-R= mw dd) Now, if Am be the mass, to be dumped, then using the Eq. F, = mw, R-(m- Am) g= (m-Am)w, Q) 2mw From Eqs. (1) and (2), we get, Am= a0 Let us write the fundamental equation of dynamics for all the three blocks in terms of projections, having taken the positive direction of x and y axes as shown in Fig; and using the fact that kinematical relation between the accelerations is such that the blocks move with same value of acceleration (say w) my &-T,= myw Q) T, - T, - kn, g=m,w (2) and T,-kmg= mw (6) The simultaneous solution of Eqs. (1), (2) and (3) yields, [img - k (my + my) ] | —memem =F (1+k) my ona oo ams ‘Mog As the block mg moves down with acceleration w, so in vector form pe (mto= kom + ma NE Mg + my +m, Let us indicate the positive direction of x-axis along the incline (Fig.). Figures show the force diagram for the blocks. Let, R be the force of interaction between the bars and they are obviously sliding down with the same constant acceleration w. 36 1.62 Newton’s second law of motion in projection form along x-axis for the blocks gives : m, g sin a — ky m, g.cos..+R= m,w Q) m,g sina —R-k,m,g.cosa= m,w 2) Solving Eqs. (1) and (2) simultaneously, we get km, + ham, : my +m, m, m, (k, - ke) g cos a m, +m, (b) when the blocks just slide down the plane, w= 0, so from Eqn. (3) km + km, 2 m, +m, or, (m,+ m))sina = (k,m,+k,m,) cosa (km, + kam) m+ im, Case 1. When the body is launched up : Let k be the coefficeint of friction, u the velocity of projection and / the distance traversed along the incline. Retarding force on the block = mg sin a +k mg cos a and hence the retardation = gsina +kgcos a. Using the equation of particle kinematics along the incline, O= w-2(gsina+kgcosa)! 2 w= gsina-gcosa ind Re @) gsina-gcos a 0 Hence tana, 2 '" gana vkgce a) @ and O= u~(gsina+kgcosa)t or, us gsina +kgcos.a)t @ Using (2) in (1) T= F sin +k gcosa)s? e Case (2). When the block comes downward, the net force on the body = mg sin a. - kn g cos and hence its acceleration = g sin a - kg cos @ Let, t be the time required then, In $ @sin ok gcosa) Oy From Eggs. (3) and (4) ne _ sina -koosa r? sina+kcosa But = 5 (according to the question), Hence on solving we get 2 ke ya punae 0-16 + 1.63 1.64 1.65 37 At the initial moment, obviously the tension in the thread connecting m, and m, equals the weight of m,. :(a) For the block m, to come down or the block m, to go up, the conditions is m,g-Tz0 and T-m,gsina-fre0 where 7 is tension and f is friction which in the limiting case equals km,g cosa. Then or = m,g—mysina>km,g cosa A 72, (kcos.a.+ sin a) m (b) Similarly in the case m, 8 sin & — m8 > frinn or, m, gsin a - mg > km, g cosa 7 7 < ina-ko0s.a) m (©) For this case, neither kind of motion is possible, and fr need not be limiting. m, Hence, (cos ot + sin a) > => (sin a - k cos a) 1 From the conditions, obtained in the previous problem, first we will check whether the mass m, goes up or down. Here, m,/m, = 1 > sin a + kcos a, (substituting the values). Hence the mass m, will come down with an acceleration (say w). From the free body diagram of previous problem, Ma, 8 ew, @) and T-m,gsina- km, g cos. a= m,w (2 ‘Adding (1) and, (2), we get, Mm, g-m,gsina-—km,gcosa= (m,+m)w wa Ltz/m—sina—koos a) 8 _ (ny - sin a —k eos a) g (1 +m,/m,) 1+H Substituting all the values, w= 0-048 g ~0-05 g ‘As m, moves down with acceleration of magnitude w= 0.05 g>0, thus in vector form acceleration of m,: >. (-sina-ke0s a) 2" 990 w, Tea 0.05 g° Let us write the Newton’s second law in projection form along positive x-axis for the plank and the bar fr= mw, fr= mw, qa) 38 1.66 1.67 At the initial moment, fr represents the static friction, and as the force F grows so does the friction force fr, but up to it’s limiting value fue. f= frauen) KN= km, g. frm wk Unless this value is reached, both bodies moves Fp as a single body with equal acceleration. But as soon as the force fr reaches the limit, the bar starts sliding over the plank i.e. w.2 wy. Substituting here the values of w, and w, taken from Eq, (1) and taking into account that f, = km, g,we obtain, (at - km, g)/m, 2 ws g, were the sign "=" corresponds to the moment 1 t= ty (ay) k + Hence, to - eee eeelee. ma (my m™) am, If tty then w, = 22 (constant), and a Ww, = (at- km, g)/m, On this basis w, (¢) and w, (2), plots are as shown in the figure of answersheet. Let us designate the x-axis (Fig.) and apply F,= mw, for body A : mgsina-kmgcosa= mw or, we gsina-kgcosa Now, from kinematical equation : Isec a = 0+ (1/2) we? or, t= V2Tsec a/(sin a — k cos G) g N = V21/(6in2 a/2 - keos*a) g LV (using Eq. (1)). a(- kena) for tain» 4a 0 A — ie. 20826, 2keosasina= 0 or, un2a= -t-a= 49° and putting the values of a, k and / in Eq. (2) we get fai, 18. Let us fix the x-y co-ordinate system to the wedge, taking the x-axis up, along the incline and the y - axis perpendicular to it (Fig.). 39 Now, we draw the free body diagram for the bar. Let us apply Newton’s second law in projection form along x and y axis for the bar : Tcos B-mgsina-fr= 0 (1) TsinB+N-mgcosa= 0 or, N= mgcosa-TsinB (2) But f, = kN and using (2) in (1), we get T= mgsina +k mg cos o/(cos f + k sin B) @) For T,,,, the value of (cos B + ksin B) should be maximum d (cos B + ksin B) So, aB =0 or tanpek Putting this value of B in Eq. (3) we get, po —mgsina+keosa) __ mg (sina +kcos a) Ma /Viek? +?/Vi eke View First of all let us draw the free body diagram for the small body of mass m and indicate x-axis along the horizontal plane and y — axis, perpendicular to it, as shown in the figure. Let the block breaks off the plane at t= % ic. N= 0 So, N= mg-aysina= 0 oye ana a) From F, = mw,, for the body under investigation : md y/dt= atcos a. ; Integrating within the limits for v(t) v mf av, cos. frat (using Eq. 1) 0 o ds aos & 2 - on ® Integrating, Eqn. (2) for s (t) 3 sn Seer Q) Using the value of f= fy from Eq. (1), into Eqs. (2) and (3) mg’ cosa m? g* cos a. ve Sige ANd $e 2asin’a 6a’ sin’ a 40 1.69 1.70 L7L Newton’s second law of motion in projection form, along horizontal or x-axis i.e. F,= mw, gives. F cos (as) = my (as a= as) or, Fcos (as) ds = mvdv Integrating, over the limits for v (s) E ¥ se coon de a or ve Vv ma = V2gsina/3a (using F= ™) which is the sought relationship. From the Newton’s second law in projection from : For the bar, T-2kmg = (2m)w @ For the motor, T- kang = mw’ (2) Now, from the equation of kinematics in the frame of bar or motor : l= Sows wys @) From (1), (2) and (3) we get on eliminating T and w t= V2i/(kg+3w) Let us write Newton’s second law in vector from F'= mw, for both the blocks (jin the frame of ground). T+ mg mW @ Ts m3 maw @ These two equations contain three unknown quantities #,, W, and T. The third equation is provided by the kinematic relationship between the accelerations : a ee Wis WotW , Wye WH 3) where Wis th acceleration of the mass m, with respect to the pulley or elevator car. Summing up termwise the left hand and the right-hand sides of these kinematical equations, we get 172 41 Wi = 25% @ The simultaneous solution of Eqs+(1), (2) and (4) yields > = ap. =m) 2m, , my, m2 Using this result in Eq. (3), we get, —y _ MyM >» —. wz 2mm, _, . m, +m, — Wo) and T= m, +m. (H - 8) cS fe —»~ My, M, > >. Using the results in Eq. (3) we get W = (e- Wo) m, +m, (6) obviously the force exerted by the pulley on the celing of the car m, +m, Note : one could also solve this problem in the frame of elevator car. Let us write Newton’s second law for both, bar 1 and body 2 in terms of projection having taken the positive direction of x, and x, as shown in the figure and assuming that body 2 starts sliding, say, upward along the incline T,-m,gsina= mw, () ae N m,g-T,= m,w Q) Ti For the pulley, moving in vertical direction Ti from the equation F, = mw, 2T,-T,= (m,)w,= 0 of "mg. 2 | (as mass of the pulley m,= 0) oo or T,= 27, @) As the length of the threads are constant, the 29, kinematical relationship of accelerations becomes w= 2m, (4) Simultaneous solutions of all these equations yields : m | 25(2%2-se] 2g(29 in al) w= ——___—_+ m. 4n+1 (<2) ae) m, As n> 1, w is directed vertically downward, and hence in vector form we 22(2y-sina) 4y+1 42 1.73 1.74 Let us write Newton’s second law for masses m, and m, and moving pully in vertical direction along positive x - axis Fig.) : m, 8-T= mw es m,g-T= mW, 2 T,-2T= O(asm= 0) or T,=2T @) Again using Newton’s second law in projection form for mass my along positive x, direction (Fig.), we get T= my 4 The kinematical relationship between the accelerations of masses gives in terms of projection on the x - axis Wig t Wo = 2H () Simultaneous solution of the obtained five equations yields : a [4 m, mz + mg (m, - m,) 18 : 4 mm, +m (m, ie m) In vector form ap = mim + mo (m,- 18" 1 4m, m, + my (m, +m) As the thread is not tied with m, so if there were no friction between the thread and the ball m, the tension in the thread would be zero and as a result both bodies will have free fall motion. Obviously in the given problem it is the friction force exerted by the ball on the thread, which becomes the tension in the thread. From the condition or language of the problem w,,>Ww, and as both are directed downward so, relative acceleration of M = w,,-w,, and is directed downward. Kinematical equation for the ball in the frame of rod in projection form along upward direction gives : b= 4 Gy — Wy)? @ Newton’s second law in projection form along Tz fi vertically down direction for both, rod and ball ele gives, Mg -fr= Mwy Q) fr mg fr= mq ® wp ale Multiplying Eq. (2) by m and Eq. (3) by M m and then subtracting Eq. (3) from (2) and after ¢ using Eq. (1) we get t 21Mm PO Oteme "4 LIS 1.76 43 Suppose, the ball goes up with accleration w, and the rod comes down with the acceleration w. As the length of the thread is constant, 2 w, = w, ro) From Newton’s second law in projection form along vertically upward for the ball and vertically downward for the rod respectively gives, T-mg= mw, Q) and Mg-T'=Mw, (3) but T=2T (because pulley is massless) (4) From Eqs. (1), (2), (3) and (4) _ QM-m)g_ (2-neg Me n+ 4M +4 acre (downwards) From kinematical equation in projection form, we get (in upward direction) and w)= 1 2 Is z (w, +w,)t as, w, and w, are in the opposite direction. Putting the values of w, and w,, the sought time becomes t= V21@+4)/3Q2-n)g = 14s Using Newton’s second law in projection form along x-axis for the body 1 and along negative x - axis for the body 2 respectively, we get mg -T,= m,™, @) T,~m, = mW, @ For the pulley lowering in downward direction from Newton’s law along x axis, T,-2T, = 0 (as pulley is mass less) or, T,= 27, (3) As the length of the thread is constant so, wy = 20, (4) The simultaneous solution of above equations yields, 2(m,-2m)g 2-2) my, oe ee AM) as 5 ea ee i ®) Obviously during the time interval in which the body 1 comes to the horizontal floor covering the distance h, the body 2 moves upward the distance 2h. At the moment when the body 2 is at the height 2h from the floor its velocity is given by the expression : Ba 2, (2n)= 2{ 2M=DE] 94. SA 2) 14 ne4 After the body m, touches the floor the thread becomes slack or the tension in the thread zero, thus as a result body 2 is only under gravity for it’s subsequent motion. 44 LH 1.78 Owing to the velocity v, at that moment or at the height 24 from the floor, the body 2 further goes up under gravity by the distance, 2g nt+4 Thus the sought maximum height attained by the body 2 : Ho 2heh' = thy M=2), Sah (™m+4) n+4 Let us draw free body diagram of each body, i.e. of rod A and of wedge B and also draw the kinemetical diagram for accelerations, after analysing the directions of motion of A and B. Kinematical relationship of accelarations is : Wa tana= —* man @) Let us write Newton’s second law for both bodies in terms of projections having taken positive directions of y and x axes as shown in the figure. m,g-N cos a= my wy Q) and Nsin a= mg Wy (3) Simultaneous solution of (1), (2) and (3) yields : | m,gsina : 2 and ‘4 m,Sin a+ mg cotacosa (1+ cot” a) Wa =—4. —_-8____ Wa" tana (lana +n cota) N A We ve NE = A } NN Wy VA Mag e We Note : We may also solve this problem using conservation of mechanical energy instead of Newton’s second law. Let us draw free body diagram of each body and fix the coordinate system, as shown in the figure. After analysing the motion of M and m on the basis of force diagrams, let us draw the kinematical diagram for accelerations (Fig.). As the length of threads are constant so, spy = dsy and as Vay and Vy, do not change their directions that why |Fae | = [Phe | = w Gay) ana Wr tt Vy and Wy tt Dy 179 a Wn tT? T o > N ON Ny Wm T > Wm T ™m sx 7 Wh AS W,, = Wray + Wy so, from the triangle law of vector addition Wy = V2 w @ From the Eq. F, = mw, , for the wedge and block : T-N= Mw, Q) and N= mw @) Now, from the Eq. F,= mw, , for the block mg -T~kN = mw (4) Simultaneous solution of Eqs. (2), (3) and (4) yields : a: mM ee (n+ 2m+M) (k+2+M/m) Hence using Eq. (1) (eee im" Oaks M/m) Bodies 1 and 2 will remain at rest with repect to bar A for Wain SWS Wongg) WHELE Woy iS the sought minimum acceleration of the bar. Beyond these limits there will be a relative motion between bar and the bodies. For 0 < w k Let us write Newton’s second law for both bodies in projection form along positive yp and x, axes as shown in the Fig. mm gcosa-N=mW24y,)= Ma [Waronrt rir] * m[O+Mrsine] or, m,gcosa-N-= mw, sina @ and Nsina = mw, 2) Solving (1) and (2), we get i. ma gsinacosa __ gsin a cos a m,+m,sin?a — (m,/m,)+sin?a 47 1.82 To analyse the kinematic relations between the bodies, sketch the force diagram of each body as shown in the figure. On the basis of force diagram, it is obvious that the wedge M will move towards right and the block will move down along the wedge. As the length of the thread is constant, the distance travelled by the block on the wedge must be equal to the distance travelled by the wedge on the floor. Hence ds, = dS, AS Vny and vy, do not change their directions and acceleration that’s WhY Wry tt Vegg and Wy tt Vy and Wyse = Wy = W (say) and accordingly the diagram of kinematical dependence is shown in figure. : Wy AS W,." Wry + Wy, 80 from triangle law of vector addition. Wy = Wwe + Wey 2 Wey Wy COS = wV2(1 — cos a) q@ From F, = mw, , (for the wedge), T= Toosa+Nsina= Mw Q) For the bar m let us fix (x-y) coordinate system in the frame of ground Newton’s law in projection form along x and y axes (Fig.) gives mg sino. - T= mw,, «=m [mu wot | = [Wace 008 (1-1) =m w (1 - coset) @) MmECOsa—N= mMqi5)™ [Wms * Macy) |= MLO + w sinc] (4) Solving the above Eqs. simultaneously, we get mgsina * 42m (1—cosa) Note : We can study the motion of the block m in the frame of wedge also, alternately we may solve this problem using conservation of mechanical energy. 1.83 Let us sketch the diagram for the motion of the particle of mass m along the circle of radius R and indicate x and y axis, as shown in the figure. ~ (a) For the particle, change in momentum Ap = mv (-i)-mv(j) so, [Ap P| = V2 mv and time taken in describing quarter of the circle, 48 1.84 185 xR Ate oF ay 2 Hence, =lAPL. vim W2my? —_, 4 At wR/2v mR Vp => (b) In this case Ve — ~ a p= 0 and py= mw,t(-i), . so |Ap|=mw,t = Hence, || = iezl. mw, " While moving in a loop, normal reaction exerted by the flyer on the loop at different points and uncompensated weight if any contribute to the weight of flyer at those points. (a) When the aircraft is at the lowermost point, Newton’s second law of motion in projection form F, = mw, gives 2 mv N-mg= ane my? or, N= mg +" = 2-09 KN (b) When it is at the upper most point, again from F, = mw, we get w+ mg = mv? N"= -mg= 0-7kN (c) When the aircraft is at the middle point of the loop, again from F, = mw, 2 my. a ae N’ R 1-4kN The uncompensated weight is mg. Thus effective weight = VN" + mg” = 1-56kN acts obliquely. Let us depict the forces acting on the small sphere m, (at an arbitrary position when the thread makes an angle @ from the vertical) and write equation F. = mw via projection on ea the unit vectors @, and u,. From F,= mw,, we have mg sin 0 =m & . a wm Le, yp ds 1(-d®) (as vertical is refrence line of angular position) 49 or vdv= —glsindd Integrating both the sides : v 8 fvdv= -gif sinodo 0 wd 2 v or, 7 78! cos 8 2 Hence “= 2g 00s 0= , a) (Eq. (1) can be easily obtained by the conservation of mechanical energy). From F, = mw, T-mgcos0= ar Using (1) we have T= 3 mg cos ® (2) Again from the Eq. F,= mw,: mg sin 0 = mw, or w,= gsinO Q) Hence w= Vw? + w2 = V(gsin 0)? + (2g 00s 0)” (using 1 and 3) = gV1+3.c0s76 (b) Vertical component of velocity, v, = v sin @ So, vy = v?sin?@= 2g I cos @ sin? (using 1) tn 2 For maximum v, or v2, Hivos Gin 9) Lo oo 1 which yields cos O= Therefore from (2) T= 3mg Fe = VE mg (6) We have Wa W,ti,+ Wy tly thus Wy = Way) + Wy) But in accordance with the problem w, = 0 So, Way) + Wy = or, g sin @ sin 0 + 2g cos 70 (- cos 8) = 0 1 or, cosO= 7 or, O= 547° 50 1.86 1.87 The ball bas only normal acceleration at the lowest position and only tangential acceleration at any of the extreme position. Let v be the speed of the ball at its lowest position and / be the length of the thread, then according to the problem a= gsina (1) where ot is the maximum deflection angle From Newton’s law in projection form : F,= mw, mg sin 0 = my oe o, gl sinOd0= vdv On integrating both the sides within their limits. 0 7: foo ° : or, v= 2gi (1 -cos a) (2) Note : Eq. (2) can easily be obtained by the conservation of mechanical energy of the ball in the uniform field of gravity. From Eqs. (1) and (2) with @ = a 2gl (1 - cos a) = Ig cos a or, cosa= 2 so, a= 53° Let us depict the forces acting on the body A (which are the force of gravity mg”and the normal reaction N ) and write equation F = mw via projection on the unit vectors u, and u, Fig.) From F,= mw, . dv mg sin 8 = "ae vay ne fide «Rad or, gRsin0d0= vdv Integrating both side for obtaining v (0) e v J gRsinodo~ fvav oO 0 or, \yv? = 2gR (1 - cos 0) From F,= mw, mg cos 0- N= m= me At the moment the body loses contact with the surface, N= 0 and therefore the Eq. (2) becomes v= gR cos 0 QB) 1.88 1.89 1.90 51 where v and 8 correspond to the moment when the body loses contact with the surface. Solving Eqs. (1) and (3) we obtain cos @ = 2 or, 8= cos~* (2/3) and v= V2gR73. At first draw the free body diagram of the device as, shown. The forces, acting on the sleeve are it’s weight, acting vertically downward, spring force, along the length of the spring and normal reaction by the rod, perpendicular to its length. Let F be the spring force, and AJ be the elongation. From, F,= mw, : Nsin@+Fcos0= mw*r (ty where rcos 0 = (Ip + AD. Similarly from F,= mw, Ncos@-Fsin®= 0 ot, N=Fsin@/cos 0 2) From (1) and (2) F (sin 0/cos 8) - sin 0+ F cos 0 = mw’ r = mw (I, + Al)/cos @ On putting F= «Al, x Asin? 6 +k Al cos?0 = mw? (Iy+AD) on solving, we get, by 4 K-ma? (x/mo*—1) and it is independent of the direction of rotation. According to the question, the cyclist moves along the circular path and the centripetal force is provided by the frictional force. Thus from the equation F, = mw, 2 2 my mv fre BE or kong = ™ Al= mo? 2 io wo(t-gle- > or v= ky(r-7/R)g ) alr For Viy,y WE should have or, 1-2. 0, so r= R/2 Hencg Yigg = $V Fah As initial velocity is zero thus v= 2ws @ As w,>0 the speed of the car increases with time or distance. Till the moment, sliding starts, the static friction provides the required centripetal acceleration to the car. Thus fr= mw, but frs kmg 52 191 1.92 So, ws he or, Wake Re or, Ps @e-w)R Hence Var VR - WR so, from Eqn. (1), the sought distance s = fet es = 60m. Since the car follows a curve, so the maximum velocity at which it can ride without sliding at the point of minimum radius of curvature is the sought velocity and obviously in this case the static friction between the car and the road is limiting. Hence from the equation F, = mw 2 kang z om or vs VERg so Vmax = WE Ronin 8 + (@) We know that, radius of curvature for a curve at any point (x, y) is given as, [1+ @/aey? ¥? Re énla? For the given curve, Yy 4 og (*) and Y= ~% gin = a ala oo Substituting this value in (2) we get, : [1 + (27/02) cos? (x/a) }°7 (a/c?) sin (x/a) For the minimum R, == = a and therefore, corresponding radius of curvature 2 a Rein °) Hence from (1) and (2) Vinag = OV ga The sought tensile stress acts on each element of the chain. Hence divide the chain into small, similar elements so that each element may be assumed as a particle. We consider one such element of mass dm, which subtends angle da. at the centre. The chain moves along a circle of known radius R with a known angular speed w and certain forces act on it. We have to find one of these forces. From Newton’s second law in projection form, F, = mw, we get 2Tsin (da./2) - dN cos 0 = dmwR and from F, = mw, we get aN sin® = gdm Then putting dm = mdo/2x and sin (da/2)= da/2 and solving, we get, 2 T- m (w" R + g cot ®) 2n 53 Ty " dmg : 1,93 Let, us consider a small element of the thread and draw free body diagram for this element. (@) Applying Newton’s second law of motion in projection form, F, = mw, for this element, (T+ dT) sin (d 0/2) + Tsin (d0/2)-dN= dmw’R= 0 or, 27 sin (d 0/2) = AN, [negelecting the term(dT sin d 8/2) ] _d0_ do or, Td0- AN, as sin = 5 0) Also, dfr= kdN= (T+dT)-T= dT T Q) From Eqs. (1) and (2), kTd0= aT or “Fa kao In this case Q= x so, nee 3 or, or, na x @) 7 So, ke Zin em Zinn m. (b) When —>= n, which is greater than my the blocks will move with same value of 1 acceleration. (say w) and clearly m, moves downward. From Newton’s second law in projection form (downward for m, and upward for m,) we get : m,g-T,= m,w (4) and T,-m,g= mw 6) 54 1.94 1.95 1.96 197 I, 2 Also 7,7 No ©) Simultaneous solution of Eqs. (4), (5) and (6) yields : _ = Nom) 8 _ (=m) cae (mm, +79) (M+M9) i. The force with which the cylinder wall acts on the particle will provide centripetal force necessary for the motion of the particle, and since there is no acceleration acting in the horizontal direction, horizontal component of the velocity will remain constant througout b the motion. So Vg HV COS Using, F,,= mw,,, for the particle of mass m, mvi_ mv,cos’a Ree which is the required normal force. Obviously the radius vector describing the position of the particle relative to the origin of coordinate is N= Re ee > > r= xityj= asinotitbcoswtj Differentiating twice with respect the time : 2 — We EF = ~ 02 (asin o17+ boos wrj}= ~0F7* Q) t So 2 Thus F=mw=-mor ors (a) Wehave Ap™= f Far = fmgzar= mz () 0 ~— : : . : 2(%8) (b) Using the solution of problem 1.28 (b), the total time of motion, t= - ae Hence using t= vin (1) [ap"] = mgt = -2m(wyeV/e (Wo Bis -ve) From the equation of the given time dependence force F'= a t(t-1) at t=, the force vanishes, (a) Thus app f Fa o 55 or, but Ras ye Qu ee Ge () Again from the equation F= mw av" = at(t-t)= ma or, atv-t?)dt= md” Integrating within the limits for v%), or, Thus Hence distance covered during the time interval t= +, © se fva 198 We have F = Fysin wt av ey ra m= Fysin wt or mdv™ Fysin wtdt or On integrating, a» —-F . A inly = —* cos wt + C, (where C is integration constant) = F 0 When t= 0, v=0, so C= ——~ mo a», ~Fo Ce ae eee nee See mo mo Fo As | cos wt < 1 so, v= —*-(1-cos of) mo 56 ' Thus s= fr at 0 Fyt Fosinot ‘0 : eae 3 ( ot - sin wt) mo mo mo (igure in the answer sheet). 1.99 According to the problem, the force acting on the particle of mass m is, F= Fycos ot dave = Fi So, a= Fy cos wt or dv'= — cos wt dt a m Integrating, within the limits. v ' t 3 *F F, dv= © f cos aed or v= moot oO 0 It is clear from equation (1), that after starting at t= 0, the particle comes to rest fro the first time at r= 2, ® — Fy. x From Eq. (1), v= |of= — sinc for rs = Q) Thus during the time interval t = x/c, the sought distance Fon s= 2 f sinovara a mw mo 0 From Eq. (1) Fy 7 “max” Zo | sin we | = 1 a 1.100 (a) From the problem = F= - 7 so m&P= -7v dv es Thus mae -m[asdvthv] or, wile v m On integrating Inve-7 04 But at t= 0, v=v, 80, C= Invy r tt or, -—t Of, v= yom m Thus for tao, v=0 dv =F (b) We have m7 = -rv so dv= me 1101 587 Integrating within the given limits to obtain v(s ): y or, @) Thus for (c) Let we have or, 0 0 So i =m In(1/)) | minn r r Now, average velocity over this time interval, Bin fn : f vge”m dt fra % %(n-1) ee ion ;mn According to the problem oe mn -kv ee kdt Integrating, withing the limits, v ' da k m (%-¥) fs ale ~r a % To fin. the value of k, rewrite a 2 ee ms -kv* or, , -at On integrating v h ee: v m. ' 0 m, Yo So, ke Fn Q) Putting the value of k from (2) in (1), we get A(vy-¥) t= —— Yo vyvin 58 1.102 1,103 From Newton’s second law for the bar in projection from, F, = mw, along x direction we get mg sin ot - kg cos a= mw co ve. gsinat—arg cosa, (ask= ax), or, vdv= (gsina-argcos a) de or, Jf vav= ef (sina -x cos a) dr o Oo 2 2 v x So, P= g(sinax-7 From (1) v= 0 at either cosa) (a) 2 x= 0, or x= qine As the motion of the bar is unidirectional it stops after going through a distance of 3 em . tan a. From (1), f0F Ypgys (sin ax -%acos a) = 0, which yields x= tuna a& 2 G y a Hence, the maximum velocity will be at the distance, x= tana/a Putting this value of x in (1) the maximum velocity, gsin o tan a Vinax max a Since, the applied force is proportional to the time and the frictional force also exists, the motion does not start just after applying the force. The body starts its motion when F equals the limiting friction. Let the motion start after time fg , then kan, F = aty=kmg or, ty= = So, for t= s fo, the body remains at rest and for ¢ > fy obviously id nt a(f-%) of, mdv= a(t-t)de Integrating, and noting v = Oat £ = fy, we have for t> fy ’ ‘ f mave a fe-ma or v= 5 (t=) 0 ‘y ' C 2 a 3 Thus (t-t)) dt= Sm (toto) fo 1.104 1.105 59 While going upward; from Newton’s second law in vertical direction : m *e =(mg+kv?) or —U = ~ ds *) At the maximum height h, the speed v = 0, so oO h Integrating and solving, we get, 2 m ky ha in ( 1+ =) @ When the body falls downward, the net force acting on the body in downward direction equals (mg - ky? )» Hence net acceleration, in downward direction, according to second law of motion vdv by? vdy en eo, Eye as e- Thus f en fe g-kv*/m Integrating and putting the value of h from (1), we get, ore Let us fix x - y co-ordinate system to the given plane, taking x-axis in the direction along which the force vector was oriented at the moment f= 0, then the fundamental equation of dynamics expressed via the projection on x and y-axes gives, : dy, cos Wt = 7a (i) d and Fsinot= m— Q a (@) Using the condition (0) =0, we obtain v, = —— sinws Q) and v= (1 - cost) 4 Hence, 60 1.106 1.107 (b) It is seen from this that the velocity v tums into Zero after the time interval At, which can be found from the relation, of. x. Consequentely, the sought distance, is vdt Average velocity, = os 2F_ .. (wt 4F So, <> -S% sn( 2 al (2x0) = 0 The acceleration of the disc along the plane is determined by the projection of the force of gravity on this plane F, = mg sin a and the friction force fr = kmg cos a. In our case k= tana and therefore fr= F,=mgsina Let us find the projection of the acceleration on the derection of the tangent to the trajectory and on the x-axis : mw,= F,cos @ - fr= mg sina (cos p-1) mw,= F,- frcosp= mgsina(1-cos p) Itis seen fromthis that w, = - w,, which means that the velocity v and its projection v, differ only by a constant value C which does not change with time, i.e. va y+, where v,= vcos @. The constant C is found from the initial condition v= vo, whence C= vp since p = 5 initially. Finally we obtain v= vy/(1+cos@). In the cource of time @—> 0. and v—> vp/2. (Motion then is unaccelerated.) Let us consider an element of length ds at an angle @ from the vertical diameter. As the speed of this element is zero at initial instant of time, it’s centripetal acceleration is zero, and hence, dN - dds cos p= 0, where 2 is the linear mass density of the chain Let T and T+dT be the tension at the upper and the lower ends of ds. we have from, F,= mw, (T+ dT)+)ds gsing-T= Msw, or, aT+Rdg gsing = dds, 1.108 61 If we sum the above equation for all elements, the term f dT = 0 because there is no tension at the free ends, so VR rer f sinpdg- dw, f ds=Alw, 0 Hence w,= (10s 1 R As w, =a at initial moment Sek So, w=|w,|=2F (1 - cos 5 In the problem, we require the velocity of the body, realtive to the sphere, which itself moves with an acceleration Ws in horizontal direction (say towards left). Hence it is advisible to solve the problem in the frame of sphere (non-inertial frame). At an arbitary moment, when the body is at an angle @ with the vertical, we sketch the force diagram for the body and write the second law of motion in projection form F,= mw, my? or, mg cos 0 - N ~ mw sin @ =" qa At the break off point, N= 0, @= 0, and let v= vgso the Eq, (1) becomes, 2 Yo i. FF ™ F.08 Op ~ Wo sin Oy 2) From, F,= mw, img sin 0 — mw cos = m2 w= m 2X ds ~ "RdO or, vdv= R(gsin 0 +w, cos 0)d0 ® % Integrating, f vdv= fi R (gsinO + wy cos0) dO 0 Oo (1 - c080,) + Wo sin Oy @) Note that the Eq. (3) can also be obtained by the work-energy theorem A= AT (in the frame of sphere) therefore, mgR (1 ~ cos Op )+ mip sin 0) = 3 mv [bere mw) sin Q, is the work done by the pseudoforce (- mw,)] 2 Yo 7 or, FH 7 8 (1 ~€05 0p) + wy sin Oy 62 1.109 1.110 Solving Eqs. (2) and (3) we get, 4 2 Vp = V2gR73 and 05 = cos ~* PALA EEL ie where k= “2 3(1 +k) 8 Hence % Layee 17? This is not central force problem unless the path is a circle about the said point. Rather here F, (tangential force) vanishes. Thus equation of motion becomes, V,= Vo* constant 2 myo A and, —. = for r= ry i r We can consider the latter equation as the equilibrium under two forces. When the motion is perturbed, we write r= ry) +x and the net force acting on the particle is, 2 2 2 my i mv mv -—4A—+ 0 ~=4 1-%),22(1-2)_-=2a-ayz (r+x)" TotX rg i) To 1% my, This is opposite to the displacement x, if n< 1+ (tet is an outward directed centrifugul force while —4 is thé inward directed external force). r There are two forces on the sleeve, the weight F, and the centrifugal force F,, We resolve both forces into tangential and normal component then the net downward tangential force on the sleeve is, 2. mg sin® (: - Roos 4) This vanishes for 0= 0 and for oO @= 0) cos”? a} which is real if wR 2 2R Siz oR>g. IfwR g, 1--—~ cos0 is negative for small gsn0" mg=F; mg CosO+mu*RSin® © near 0 = 0 and @= 0 is then unstable. However 0 = Q, is stable because the force tends to bring the sleeve near the equilibrium position @ = @,, If wR = g, the two positions coincide and becomes a stable equilibrium point. 11 1112 1113 63 Define the axes as shown with z along the local vertical, x due east and y due north. (We assume we are in the northern hemisphere). Then the Coriolis force has the components. F.,= -2m@xV) = 2meo | v, cos® ~ v, sin®) i= v, cos j+v, cos k] = 2m (v, cos0 — v_ sind) 7” since v, is small when the direction in which tie gun is fired is due north. Thus the equation of motion (neglecting centrifugal forces) are X' = 2mo (v, sing - v, cos), y= 0 and Z = -g -North Integrating we get y= v (constant), z= - gt z-vertical and X= 2w vsing t + wg t? cosp East Finally, > 7 x= ov? sing +3 gt? cose Now v >> gt in the present case. so, 2 2 x~ ovsing (=) = wsing = v v ~ 7m (to the east), The disc exerts three forces which are mutually perpendicular. They are the reaction of the weight, mg, vertically upward, the Coriolis force 2mv' w perpendicular to the plane of the vertical and along the diameter, and ma’r outward along the diameter. The resultant force is, Famv g+a'r+(Qv a) The sleeve is free to slide along the rod AB. Thus only the centrifugal force acts on it. The equation is, 3 dr mi= mo?r where v= &- a Butp= v%— #(2 *) dr dr s0, ive 50??? + constant 7 Pa ke? Vo being the initial velocity when r= 0. The Coriolis force is then, amo Vig raPF = ama Viva = 2-83 N on putting the values. 64 1.114 1115 The disc OBAC is rotating with angular o! velocity w about the axis OO’ passing through the edge point O. The equation of motion in w rotating frame is, mi’ = F's mo? Rs Inv’ x= F+F, 8 where F,, is the resultant inertial forc. (pseudo force) which is the vector sum of centrifugal 0 A and Coriolis forces. (a) AtA,F,, vanishes. Thus 0= -2mw?Rin+2mv' on where 7 is the inward drawn unit vector to the centre from the point in question, here A. Thus, v= oR 2 2 =e He gt so, w RO? R. > 2 aot (b) AtB Fi,= ma? OC + mo BC its magnitude is mo? VaR? —??, where r= OB. The equation of motion in the rotating coordinate system is, > => mW = F+ mo R+2m (vx a) Now, v= RO@+Rsind pe, and = W = w'cosO@--w' sinO a > > > > & 6 ce. Fee ™ 0 RO Rsin@p 1 @ cos @ -wsin® 0 = 2 (oR sin’0 @) + wR sin 0 cos 0 @ & — WR 0 cos 0 2, Now on the sphere, Ve (-RO-R sin? 6g?) a + (RO -Rsin B cos 0") ep + (Rsin Op + 2R cos 60g) & Thus the equation of motion are, m(-R@-Rsin? 0g”) = N- mg cos 0 + mw” R sin’ @ + 2mw R sin? 6p m(R 0° -Rsin 8 cos 0 ¢*) = mg sin 6 + mw? R sin 0 cos 8 + 2mu R sin 8 cos 8p m(R sin 0 +2R cos 0 0G) = -2 mw RO cos 0 From the third equation, we get, p= -@ A result that is easy to understant by considering the motion in non-rotating frame. The eliminating @ we get, : mR @ = mg cos @-N mRO = mgsinO Integrating the last equation, mR = mg (1 - cos) 1.116 LT 65 Hence N= (3-2cos 0) mg So the body must fly off for @= @,= cos” *2, exactly as if the sphere were nonrotating. Now, at this point F,,= centrifugal force = mR sin Oy = vi mo?R Fogg = Vo RO cos” 0 + (wR) sin’ @ x 2m =V 22 RP +e Rx 4x 2k 22 age V FW RP aR x 5x Gk xm = 5 mo’ Ry s+ (a) When the train is moving aiong a meridian only the Coriolis force has a lateral component and its magnitude (see the previous problem) is, 2mwvcos 8 = 2mw sind (Here we have put R 0» v) 2n_ 54000, v3 86400 3600 ~ 2 = 3-77KN, (we write 4 for the latitude) (b) The resultant of the inertial forces acting on the train is, So, Frgerat ™ 2% 2000 x 10° x a SEAN Feor \ \) (We write X for the latitude here) Thus the train must move from the east to west along the 60" parallel with a speed, 1 te ee qoRoosh= 7x 954% 10 x 6:37 x 10 115-8 m/s = 417 km/hr We go to the equation given in 1.111. Here v, = 0 so we can take y= 0, thus we get for the motion in the xz plane. x = —2wv,cos 8 F,,= - 2mu R 6 cos 8 & + (mu? R sin 0 cos 6 + 2m w R sin 0 cos 0p) & + (mo? R sin? 0 + 2m w R sin? 0p) 2 This vanishes if = 0, = -Fo 1 . 1 Thus Vm Vy bos Ye™ ~7oRsind= -~>wRcosr and imng ; ia Integrating, aa 5 ze = wg cos gt? V2 1 a ‘2h So x= zwgcos pt zogcosp : ae eey 3 SPV There is thus a displacement to the east of 2,2 as 3" 64 500x 1x V2 26 em, 66 13 1.118 1119 1.120 1121 Laws of Conservation of Energy, Momentum and Angular Momentum. As Fis constant so the sought work done A= Fare F-@3-7) > Pat Som > ke or, A= (3i4+4j )-[(2i-3j )-G42j )]= Git4j )-(i-5j )= 173 Differentating v (s) with respect to time dv__a ds a @ Pe sO eo (As locomotive is in unidrectional motion) 2 Hence force acting on the locomotive F = mw = os Let, at v= Oat ¢ = 0 then the distance covered during the first t seconds =w Lelie @ 2 Sa gMa aaa ot 2 4242) 4.2 Hence the sought work, A = Fs = mas (et ooo 2°44 8 ‘We have 1 2. 2as* Ts gama as or, a (eh) Differentating Eq. (1) with respect to time 4as 2as 2vw= Sy on, w= Q) Hence net acceleration of the particle 7 we Vidaw = (ea) +(222) - 28Vi5 Gray Hence the sought force, F = mw = 2asV1+ (/RY Let Fi makes an angle @ with the horizontal at any instant of time (Fig.). Newton’s second Jaw in projection form along the direction of the force, gives : F = kang cos 0 + mg sin 0 (because there is no acceleration of the body.) =, > As F tt d7'the differential work done by the force F, dA= F-d7= Fds, (where ds = | d7"|) iN = kang ds (cos 0) + mg ds sin® = kmg dx + mg dy. 1 h Hence, A= kg f de+me f ay 0 0 ad = kngl+mgh= mg (kI +h). 67 1122 Let s be the distance covered by the disc along the incline, from the Eq. of increment of ME. of the disc in the field of gravity : AT+ AU = Aj, ry 0+ (- mgs sin a) = — kang cos as - king I kl - 5" Sna-keosa a Hence the sought work Ag, = ~ kg [s cos 0 +1] _ king ‘ Ag ~7 get [Using the Eqn. (1)] On puting the values A, = -0.05 J 1123 Let x be the compression in the spring when the bar m, is about to shift. Therefore at this moment spring force on m, is equal to the limiting friction between the bar m, and horizontal floor. Hence xx= km,g [where « is the spring constant (say)] (1) For the block m, from work-energy theorem : A= AT = 0 for minimum force. (A here indudes the work done in stretching the spring.) so, Fr-hn2—kngx= 0 or k5a F-kmg Q, From (1) and (2), m ro tele’) 1.124 From the initial condition of the problem the limiting fricition between the chain lying on the horizontal table equals the weight of the over hanging part of the chain, i.e. Anlg= kA(1-%) lg (where 4d is the linear N mass density of the chain) — So, ea q@ Fs Let (at an arbitrary moment of time) the length ‘of the chain on the table is x. So the net friction force between the chain and the table, at this Ax moment : 3 Adx) f,= AN= kixg 2) 4 The differential work done by the friction forces : dha Fd ~f,ds~ -kaxg-d)= de( 7h \eae ® (Note that here we have written ds = -dr., because ds is essentially a positive term and as the length of the chain decreases with time, dx is negative) Hence, the sought work done o - a a en; pe A Sf re poee (=n) ne -133 Q-n)! 68 L125 1.126 1.127 The velocity of the body, ¢ seconds after the begining of the motion becomes v= ¥> +g. The power developed by the gravity (m g’) at that moment, is P= mg" v= m(g”- v9 + 8°t) = mg (gt- vy sina) qa) As mg” is a constant force, so the average power A_ mg? ar”

= oe —— tT where Ar” is the net displacement of the body during time of flight. As, mg"L Ar” so

= 0 ‘We have ee at?, ot, v= VaRt, t is defined to start from the begining of motion from rest. So, 0,2 Ba VaR Instantaneous power, PaF y= m (Ww, le, + Wy iy, )- (VaR th, ), (where ii, and i, are unit vectors along the direction of tangent (velocity) and normal respectively) So, P= mw,VaR t= ma Rt Hence the sought average power ' ' f Pdt f ma Rt dt

e SB ' t fe 0 _maRt? | maRt 2t 2 Let the body m acquire the horizontal velocity vy along positive x — axis at the point O. (a) Velocity of the body t seconds after the begining of the motion, ve twee (vy—ket)im q@ Instantaneous power P= Fv (- longi): (vy kgt) i= — kang (vo - ket ) From Eq. (1), the time of motion t= v9/kg Hence sought average power during the time of motion 1 Hence kang vo Jf He 75 — bee ya ee : F 7 ~2W (On substitution) From F,= mw, — kg = mw, = ee or, v,dv,= -kgdx = - agxdx 1128 1.129 1.130 69 To find v (x), let us integrate the above equation v x fue - ag f xae or, Y= y-age a % 0 Now, P-F vt -maxgVve-agx? (2) For maximum. power, £We = hex") = 0 which yields x= Putting this value of x, in Eq. (2) we get, Po im Ava Centrifugal force of inertia is directed outward along radial line, thus the sought work 7 Yo v20g A= f mw? r dr = 5 mo? (3- 1) = 0-20T (On substitution) n Since the springs are connected in series, the combination may be treated as a single spring of spring constant. Kim Ky +k, From the equation of increment of ME, AT+AU=A,, 2| Ky, +K, First, let us find the total height of ascent. At the beginning and the end of the path of velocity of the body is equal to zero, and therefore the increment of the kinetic energy of the body is also equal to zero. On the other hand, in according with work-energy theorem AT is equal to the algebraic sum of the works A performed by all the forces, i.e. by the force F and gravity, over this path. However, since AT= 0 then A= 0. Taking into account that the upward direction is assumed to coincide with the positive direction of the y-axis, we can write O+dK A= A, oF 4-3 a Jar hb h Am f @ena-a 7 fer,-me ay o ° = mg f (1-209) dy= mgh 1 at) = 0. ° whence h= 1/a. The work performed by the force F over the first half of the ascent is M2 wr Ap= f Fay ame fay) a= 3 mg/4a. 0 0 The corresponding increment of the potential energy is AU = mgh/2 = mg/2a. 70 1131 1.132 From the equation F, = -Z we get F,= [-F 3] ee (a) we have at r= rq the particle is in equilibrium position. i.e. F,= 0 $0, ro= To check, whether the position is steady (the position of stable equilibrium), we have to satisfy 2 oreo d’U_[6a_ 2 We have oe | 2a Putting the value of r= ry= 3 we get 2 4 ie &. (as a and b are positive constant) au e So, ae eae ae a dr? 8a® which indicates that the potential energy of the system is minimum, hence this position is steady. (0) We have Fe -@. -3+3] dF, For F, to be maximum, —"= 0 dr 3a So, r= Sand then Frigas)” 573) As F, is negative, the force is attractive. (a) We have au -au Bo ~Gie -2ax and Fe =F = - apy So, Fe 2axi-2Byiand, Fe 2Voerepy (Q) For a central force, 7 F= 0 Here, 7xF= (eityj )x(-2axi-2Byj) = -2Bpxyk-2axy(k)«0 Hence the force is not a central force. ()) As U= ox’ + py? So, F,= z. -2ax and F,= =22 ay = -2By. So, FeVF+F=V4ers4py According to the problem F=2Va7x'+*y? = C (constant) h133 114 1.135 71 oe or, a+ py = > 2% 2. e 272 op? Therefore the surfaces for which F is constant is an ellipse. For an equipotential surface U is constant. or, = k(say) (2) So, ax’ +By= Cy (constant) oe oo or, ve" Var ap Ky (constant) Hence the equipotential surface is also an ellipse. Let us calculate the work performed by the forces of each field over the path from a certain point 1 (x,, y,) to another certain point 2 (x,, y2) gt @) dA= F-dF= ayivd7™ ayde ot, Ana f ya a (il) dA= F-d7™ (tit byi)-d7™ axde + bydy a Ya Hence Aa f axdesf byay a. n In the first case, the integral depends on the function of type y (x), i.e. on the shape of the path. Consequently, the first field of force is not potential. In the second case, both the integrals do not depend on the shape of the path. They are defined only by the coordinate of the initial and final points of the path, therefore the second field of force is potential. Let s be the sought distance, then from the equation of increment of M.E. AT+AU= Ay, ( ~ mb) + ome ssina) = - kang cosas > YO fps or, se 3g (sin a + koos a) - km, Hence Ay = ~keng 008 8 5 ay Velocity of the body at height /, v, = V2g(H— A), horizontally (from the figure given in the problem). Time taken in falling through the distance A. tzV . (as initial vertical component of the velocity is zero.) Now s= v,t= Vigi+h V2 - a= 1.136 1.137 For Seay 4 (Hh 12) =0, which yields h= # Putting this value of h in the expression obtained for s, we get, Smax = H To complete a smooth vertical track of radius R, the minimum height at which a particle starts, must be equal to Sr (one can proved it from energy conservation). Thus in our problem body could not reach the upper most point of the vertical track of radius R/2. Let the particle A leave the track at some point O with speed v (Fig.). Now from energy conservation for the body A in the field of gravity : me[h-3C +sino)]- Sm? or, v= gh(1-sin@) qa From Newton’s second law for the particle at the point O; F, = mw, , lam (h72) But, at the point O the normal reaction N= 0 N+mg sin 0 = So, ve sin 8 (2) From (3) and (4), sin 0 = 2 and v= ve After leaving the track at O, the particle A comes in air and further goes up and at maximum height of it’s trajectory in air, it’s velocity (say v’) becomes horizontal (Fig.). Hence, the sought velocity of A at this point. ¥ = ve0s (90 0)= vsind = 2+ Let, the point of suspension be shifted with velocity v, in the horizontal direction towards left then in the rest frame of point of suspension the ball starts with same velocity horizontally towards right. Let us work in this, frame. From Newton’s second law in projection form towards the point of suspension at the upper most point (say B) : 2 2 myj mvj mg+T= >" or, T= 78 (a) Condition required, to complete the vertical circle is that T= 0. But (2) 5 om = mg (2 +3 mii So, ve= vi-4gl @) 1.138 1.139 73 From (1), (2) and (3) pe MAR 48!) 7 —mgz0 of Wz v5 gl Thus Ya (ain = V5 gt From the equation F, = mw, at point C me T=—t 4 Again from energy conservation Sri = Lv? + met (6) From (4) and (5) T= 3mg Since the tension is always perpendicular to the velocity vector, the work done by the tension force will be zero. Hence, according to the work energy theorem, the kinetic energy or velocity of the disc will remain constant during it’s motion. Hence, the sought time t= & where s is the total distance traversed by the small disc during it’s motion. 0 Now, at an arbitary position (Fig.) ds = (Iy-R0)d0, UR s0, s=f a-k 8)d0 0 B RG "Ro 2R? RR or, s sia 6 “ IRV It should be clearly understood that the only uncompensated force acting on the disc A in this case is the tension T, of the thread. It is easy to see that there is no point here, relative to which the moment of force T is invarible in the process of motion. Hence conservation of angular momentum is not applicable here. Hence, the required time, Suppose that AI is the elongation of the rubbler cord. Then from energy conservation, AU, + AU, = 0 (as AT= 0) or, = mg (1+ A) + 2x Al = 0 or, 5x 1? — mg Al mg! =0 74 1.140 1.141 mgt Vf (mg) +4xSmgl 5 r] ee Cr 2x0 or, Al = xS= 147 1278 2 Since the value of V1 + a is certainly greater than 1, hence negative sign is avoided. — mel, ./ za | So, ue (1, 14 ng When the thread FA is burnt, obviously the speed of the bars will be equal at any instant of time until it breaks off. Let v be the speed of each block and 8 be the angle, which the elongated spring makes with the vertical at the moment, when the bar A breaks off the plane. At this stage the elongation in the spring. Al= hy sec @ ~ hy = ly (sec 8 ~ 1) Q) Since the problem is concerned with position and there are no forces other than conservative forces, the mechanical energy of the system (both bars + spring) in the field of gravity is conserved, ic. AT+ AU = 0 So, 2 G*) +413 (6ec0-1)?— mgiptan 0 = 0 @) From Newton’s second law in projection form N along vertical direction : 8 mg = N +x Iy (sec @ - 1) cos @ Klo Gxo-t But, at the moment of break off, N = 0. r Hence, x J, (sec 8-1) cos 8 = mg K1y)- mg or, cos 0 = ce @) "™, a 78, simultaneous solution\of (2) and (3) yields : 3 fo 19 ve = 17m/s. 32 Obviously the elongation in the cord, Al= /, (sec 6 - 1), at the moment the sliding first Starts and at the moment horizontal projection of spring force equals the limiting friction. So, x, Alsin 8 = KN qa (where x, is the elastic constant). KAL From Newton’s law in projection form along vertical direction : k, Al cos 0 +N = mg. or, N= mg~x, Alcos® From (1) and (2), (2) x, Alsin @ = k(mg - «x, Al cos @) mg 1.142 1.143 75 [oe ° 1” Aisin 0 +k Al cos 0 From the equation of the increment of mechanical energy : AU+ AT = Ay, 1 or, (Fua?)= a, king Al? ZAI (in © +k cos 0) ~ kang Ip (sec 0 - 1) 2 (sin 0 - Kcos 6) Let the deformation in the spring be Al, when the rod AB has attained the angular velocity «. From the second law of motion in projection form F,= mw, . or, Ay Thus Aj, = = 0-093 (on substitution) rt mo? | KAl= ma (p+AD or, Al= 7 (lo + AD eae From the energy equation, Ajy= 5 mi? + seal? a 2 2 2 oa mol, \ 1 [ mo I? = 5mw|h+ +3K 5 2 k = mo’ mo? (+ Alf + 5x Al? On solving We know that acceleration of centre of mass of the system is given by the expression. m+ mi Wee . m, +m, Since Wi= -w, a _ (m-m) iW We qa) oi Now from Newton’s second law F'= mit, for the bodies m, and im, respectively. bof > => T+m,Z= mw, (2) and Te m,g= m= -mw, (3) Solving (2) and (3) <> _ (m-m,) 8 8 atm . 76 1.144 1.145 1.146 Thus from (1), (2) and (4), ap (mms 7 2 (mm, + my As the closed system consisting two particles m, and of m, is initially at rest the C.M. of the system will remain at rest. Further as m,= m,/2, the C.M. of the system divides the line joining m, and m, at all the moments of time in the ratio 1 : 2. In addition to it the total linear momentum of the system at all the times is zero. So, p, =p, and therefore the velocities of m, and m, are also directed in opposite sense. Bearing in mind all these thing, the sought trajectory is as shown in the figure. First of all, it is clear that the chain does not move in the vertical direction during the uniform rotation. This means that the vertical component of the tension T balances gravity. As for the horizontal component of the tension T, it is constant in magnitude and permanently directed toward the rotation axis. It follows from. this that the C.M. of the chain, the point C, travels along horizontal circle of radius p (say). Therefore we have, Tcos0= mg and Tsin0 = mo*p Thus p= £228. ogcm @ and T= 782 SN cos (@) Let us draw free body diagram and write Newton’s second law in terms of projection along vertical aud horizontal direction respectively. Neos a - mg + frsina= 0 qa froosa-Nsina = mol Q) From (1) and (2) sin a . 2 E g)= 1 SRO (~ frsin + mg) = mo mg frees a- 1.147 2 So, fr= me (sina cosa} on @) (b) For rolling, without sliding, frs kN but, N= mg cosa-mw?Isina o71 2 mg | sina + cos a = k (mg cos o— mo? I sin ct) [Using (3)] Rearranging, we get, mo? l(cosa+ksina) = (kmg cos a - mg sina) Thus wsVg(k-tana)/(1+ktana)! = 2 rad/s (a) Total kinetic energy in frame K’ is 1 —- > 1 oo T= 5m,(H-V P+ 5m, (-V) This is minimum with respect to variation in V, when or’... ey > o> FTO be mC VP +m (oy V) = 0 vy +m, V4 > my + fe mMutMm > or ie my +m, Hence, it is the frame of C.M. in which kinetic energy of a system is minimum. (b) Linear momentum of the particle 1 in the K’ or C frame ~ mM, mM, = oe 12 >_> Prem (y- Ve) = (¥-¥2) m, +m, te. my m, or, Pi= w(%y-¥2), where, p= m+, = reduced mass Similarly, Pr= w(3-%) So, [Pils [Pil= B= Wry where, v,4= |%;- 7% @) Now the total kinetic energy of the system in the C frame is mm, 2m, 2p ee1,,2-2,15_-e I? Hence To zuya= 2H |"-%| 78 1.148 1,149 1.150 To find the relationship between the values of the mechanical energy of a system in the K and C reference frames, let us begin with the kinetic energy T of the system. The velocity of the i-th particle in the K frame may be represented as ¥;= ¥; + ¥¢. Now we can write 1 te ao tT > ymin > smi (Wr+ He) (H+ VE) 1 <> = og =D med mized zm Me Since in the C frame } m;V;= 0, the previous expression takes the form T= Td mbm +5 mv? (Gince according to the problem v¢= V) Q) Since the internal potential energy U of a system depends only on its configuration, the magnitude U is the same in all refrence frames. Adding U to the left and right hand sides of Eq. (1), we obtain the sought relationship a E Etgmv As initially U= 0 = 0, so, E=T From the solution of 1.147 (b) ot T- zuly-wl, As yl zi mm Thus Te om Ct) Velocity of masses m, and m,, after t seconds are respectively. Wy = +B and I= H+ Bt Hence the final momentum of the system, = o> = oe a P= m, vy +m, Vy = m,¥, +m,V, + (mM, +m,) Bt <> oe = > = Po+met, (where, po= mv, +m, Vv, and m= m,+m,) 1 And radius vector, r= Wott 5s Wet? Cm vyt me Va)t Aone (mrmy *2% i, ao My Wit my = vot+>et’, where vo = ————— 2 m, +m, 739 1.151 After releasing the bar 2 acquires the velocity v., obtained by the energy, conservation : 1.152 1.153 dm = de? o, ye ry & (a) 2 Thus the sought velocity of C.M. [x O+m ax Vim xv vem my my (my + my) Let us consider both blocks and spring as the physical system. The centre of mass of the system moves with acceleration a = a towards right. Let us work in the frame of centre of mass. As this frame is a non-inertial frame (accelerated with respect to the ground) we have to apply a pseudo force m,a towards left on the block mand m, a towards left on the block m, As the center of mass is at rest in this frame, the blocks move in opposite directions and come to instantaneous rest at some instant. The elongation of the spring will be maximum or minimum at this instant. Assume that the block m, is displaced by the distance x, and the block m, through a distance x, from the initial positions. From the energy equation in the frame of C.M. AT+U-A,, (where A,,, also includes the work done by the pseudo forces) Here, AF=0, U=LeG, +2F and F-m,F m,F m, F (x,+2) Wea ™ mF mF latn) ———|x)+ xy m, +m, m, +m), m,+m, or, Ree ex? = nee 2m,F So, X,+X%_ = 0 or, +2" EGm+m) , 2m,F Hence the maximum separation between the blocks equals : Jy + En, +m) Obviously the minimum sepation corresponds to zero elongation and is equal to Jy (a) The initial compression in the spring Al must be such that after buming of the thread, the upper cube rises to a height that produces a tension in the spring that is atleast equal to the weight of the lower cube. Actually, the spring will first go from its compressed 80 1.154 State to its natural length and then get elongated beyond this natural length. Let / be the maximum elongation produced under these circumstances. Then xl = mg @) Now, from energy conservation, FeAl mg(Al+)+F«i? ) (Because at maximum elongation of the spring, the speed of upper cube becomes zero) From (1) and (2), 2 a2 2mg.at 3g 0 on, Ale SME, = ¥ Therefore, acceptable solution of A/ equals 278 (b) Let v the velocity of upper cube at the position (say, at C ) when the lower block breaks off the floor, then from energy conservation. 1 ot Qo 2 gmv? = x (Al?~ 1?) -mg (1+ Al) (where = mg/x and Al = 778) or, w= 32 me Q) ee . mv+O_v At the position C, the velocity of CM; vo= “37 —= 5 —Let, theC.M. of the system (spring+ two cubes) further rises up to A yoy t fol 4 of CT Now, from energy conservation, Sonal 1 x 2m) ve (2m) g bry B 2 2 ' Yo vi dmg ' or, Ayg= f= te 2g 8g OK AL But, uptil position C, the C.M. of the system L has already elevated by, AYor= (Al+ 1D m+0 . 4mg 2m « Hence, the net, displacement of the C.M. of the system, in vpward direction Ayc™ Aya + A¥q= See 1 Due to ejection of mass from a moving system (which moves due to inertia) in a direction perpendicular to it, the velocity of moving system does not change. The momentum change being adjusted by the forces on the rails. Hence in our problem velocities of buggies change only due to the entrance of the man coming from the other buggy. From the 1.155 1.156 1.157 81 Solving (1) and (2), we get mv Mv Y= ‘Mom and v= M-m As wth wand tty” vw. =o Mv So, one and y= ——— (=m) 2” (M-m) From momentum conservation, for the system “rear buggy with man” (M+m)ip= mut ig) +Mig @ From momentum conservation, for the system (front buggy + man coming from rear buggy) Mip+m (Ut vp) = (M+m) vp we Mion "Mem Me +m Putting the value of Vg from (1), we get So, (+) mM_—> (t+ my (i) Let 07 be the velocity of the buggy after both man jump off simultaneously. For the closed system (two men + buggy), from the conservation of linear momentum, Miz +2m(i+v7)= 0 ed Vp = Vo+ 2m” or, oie on @) (ii) Let 0” be the velocity of buggy with man, when one man jump off the buggy. For the closed system (buggy with one man + other man) from the conservation of linear momentum : O= (M+ m0" +m") (2) Let v3 be the sought velocity of the buggy when the second man jump off the buggy; then from conservation of linear momentum of the system (buggy + one man) : (M+m)0" = Mvy+m(i+%)) @) Solving equations (2) and (3) we get <>» _m(2M+3m)a" y= @) (M+ m) (M+ 2m) From (1) and (4) eA +5n 1 ¥ *2@t+m) Hence v,>v, The descending part of the chain is in free fall, it has speed v= V2 gh at the instant, all its points have descended a distance y. The length of the chain which lands on the floor during the differential time interval dt following this instant is vdt. 1.158 1.159 For the incoming chain element on the floor : From dp, = F,dt (where y-axis is directed down) I ys 0- (vd) v= F, dt or F,= -Ave -2Agy Hence, the force exerted on the falling chain equals 2 v* and is directed upward. Therefore from third law the force exerted by the falling | chain on the table at the same instant of Leo time becomes Av? and is directed downward. feet Since a length of chain of weight (Ayg) already lies on the table the total force on the floor is (2dyg) + (Ayg) = (3Ayg) or the weight of a length 3y of chain. Velocity of the ball, with which it hits the slab, v= V2 gh After first impact, v = ev (upward) but according to the problem v= *, 30 e= ; Q) and momentum, imparted to the slab, = mv -(-mv’) = mv(1+e) Similarly, velocity of the ball after second impact, via ev= ey And momentum imparted = m(v/4v")= m(1+e)ev Again, momentum imparted during third impact, = m(1+e)e*v, and so on, Hence, net momentum, imparted = mv (1 +e) + mve (1 +e) + mve"(1+e) +... = mv(l+e)(ltete'+...) ae fre, (from summation of G:P) (: +4 n = V2 gh aye mV2 gh | (n + 1)/(m - 1) (Using Eq. 1) y-t (4) = 0:2 kg m/s. (On substitution) (a) Since the resistance of water is negligibly small, the resultant of all external forces acting on the system “a man and a raft” is equal to zero. This means that the position of the C.M. of the given system does not change in the process of motion. ie. 72= constant or, ArE= 0 ie. Sm, A7;= 0 or, m (AF + Aig) +M Aig = 0 > = Sar Thus, m(I"'+1)+MI= 0, or, To - o (b) As net external force on “man-raft” system is equal to zero, therefore the momentum of this system does not change, So, O- mV + ZO]+MHO 83 1.159 (a) Since the resistance of water is negligibly small, the resultant of all external forces 1.160 1.161 1.162 acting on the system “a man and a raft” is equal to aero. This means that the position of the C.M. of the given system does not change in the process of motion. ie. 7= constant or, ArZ= 0 ic. Sy m,Ar;= 0 or, m (Arnau + Ory) + M ii =-0 ml” m+M (b) As net external force on “man-raft” system is equal to zero, therefore the momentum. of this system does not change, Thus, m(I" +1)+MI= 0, ot, I= So, O= m[W" ()+BM]+M0) or, y= nee qa As ¥” () or ¥3(f) is along horizontal direction, thus the sought force on the raft Mae Mm av") dt m+M dt Note : we may get the result of part (a), if we integrate Eq. (1) over the time of motion of man or raft. In the refrence frame fixed to the pulley axis the location of C.M. of the given system is described by the radius vector ar M Aig + (Mm) Fogg tm AF r= aT) = = But Afy= ~AFay-m and AFL AFym + AFor-m mI” Thus Are Note : one may also solve this problem using momentum conservation. Velocity of cannon as well as that of shell equals V2 gi sina down the inclined plane taken as the positive x - axis. From the linear impulse momentum theorem in projection form along x - axis for the system (connon + shell) ic. Ap, = F, At: pcosa-MV2glsina = Mgsino At (as mass of the shell is neligible) pcos a-MvV 2 glsino or, At= = Mg sina From conservation of momentum, for the system (bullet + body) along the initial direction of bullet vy nvp= (m+M)¥, of, v= vom (m+M)v, of, v= ne 84 1.163 1.164 When the disc breaks off the body M, its velocity towards right (along x-axis) equals the velocity of the body M, and let the disc’s velocity’in upward direction (along y-axis) at that moment be v', From conservation of momentum, along x-axis for the system (disc + body) , , my my= (m+M)v, or Vee Se () And from energy ican for the same system in the field of gravity : Fm? dom+mv? +h mv’, +mgh , where A’ is the nem of break off point initial level. So, 1 mv 1 2 5 : zm a5 yom +M) Gt a a using (1) 2 , or, vee v- Gam -2gh' Also, if A” is the height of the disc, from the break-off point, then, ve = 2gh" So, 2g(h" +h) = 2 - ' . (+m) Hence, the total height, raised from the initial level MY = eh += 26 (M+m) (a) When the disc slides and comes to a plank, it has a velocity equal to v = V2 gh. Due to friction between the disc and the plank the disc slows down and after some time the disc moves in one piece with the plank with velocity v' (say). From the momentum conservation for the system (disc + plank) along horizontal towards right : mv m+M Now from the equation of the increment of total mechanical energy of a system ; mv= (m+M)v' or v’ 1 ot 5M +m) v?— sn? = A; 22 or, 5 t+ m) mi? = Ay (m oy 2 . 50, ts mt ~m |= as Hence, wreat eh - ugh ban un Oem reduced mass) 1.165 1.166 85 (0) We look at the problem from a frame in which the hill is moving (together with the disc on it) to the right with speed u. Then in this frame the speed of the disc when it just gets onto the plank is, by the law of addition of velocities, V = u +V2gh. Similarly the common speed of the plank and the disc when they move together is 2 m oe Then as above Ay = £(m+M)v2 - dmv? - Au? a) 2 2 a 2,_2m mm 1 Bo = 5 (m+M) fe ou uvIgh + pee ~ 5(m+M)u - 3m 2uV2gh - mgh We see that Ay, is independent of u and is in fact just - gh as in (a). Thus the result obtained does not depend on the choice of reference frame. Do note however that it will be in correct to apply “conservation of enegy” formula in the frame in which the hill is moving. The energy carried by the hill is not negligible in this frame. See also the next problem. In a frame moving relative to the earth, one has to include the kinetic energy of the earth as well as carth’s acceleration to be able to apply conservation of energy to the problem. In a reference frame falling to the earth with velocity v,, the stone is initially going up with velocity v, and so is the earth. The final velocity of the stone is 0 = v,—gt and that of the earth is v, + ae (M is the mass of the earth), from Newton’s third law, where ¢ = time of fall. From conservation of energy 2 dmg + SMvg + mgh = 2M (v + | 1 mm Hence 3% (» + ii) = mgh Negecting 7 in comparison with 1, we get va = 2gh or v, = V 2gh The point is this in earth’s rest frame the effect of earth’s accleration is of order in and can be neglected but in a frame moving with respect to the earth the effect of earth’s acceleration must be kept because it is of order one (i.e. large). From conservation of momentum, for the closed system “both colliding particles” my, + mv, = (m, +m.) 0" 3. Pd ee oT mbit mada | 1Gi-2j)+24j-6k) TF ye m, +m, 3 Hence |v] = V144+16 m/s= 46 m/s or, 86 1.167 1.168 For perfectly inelastic collision, in the C.M. frame, final kinetic energy of the colliding system (both spheres) becomes zero. Hence initial kinetic energy of the system in C.M. frame completely tums into the internal energy (Q) of the formed body. Hence oi 2 Q=T;= 2" | ¥| Now from energy conservation AT = -Q = -he jR- a In lab frame the same result is obtained as 1 (mitt mv a AT= 5 +1 2? sm |B I 2 m+m, 2 1 = =~ gH liy- BV? (a) Let the initial and final velocities of m, and m, are iy, uz and V7, V2 respectively. Then from conservation of momentum along horizontal and vertical directions, we get : my u, = mv, cos 0 Q) and my, = mv, sin ® Q tv; Squaring (1) and (2) and then adding them, w md= mid v2) Dineen Op pene Now, from kinetic energy conservation, 9 aN 1 1 1 = FMM yMmyt SMM @) ma iy or, m(uj—vi)= mvj = mate) [Using (3)] m, m, or, Wi1-— |= 2f14— my m, vs) -m: or, (:) a (4) 21) ie So, fraction of kinetic energy lost by the particle 1, Li glim ve zm 7M : ve = 1-4 jmut 4 m-m, 2m ‘ 1. - [Using (4)] ©) my+m, my+m, (b) When the collision occurs head on, mu, = my, + mV, qa) and from conservation of kinetic energy, 1.169 87 1 1 1 mus > mv +5 my 2 1 1) fm@-w) = mt gm, | mn] (Using (5)] or, v, (1+ 74} = uw, (™-1 my m ¥ (™ /m, 1) uy” (1+ m,/m,) Fraction of kinetic energy, lost 4 ‘m= my)" _4my my = 1-—t=1-(-2—4} 2 —“1"2 sing (6 aot (ism) ~gaemge CEO 6) or, (a) When the particles fly apart in opposite direction with equal velocities (say v), then from conservatin of momentum, m,u+0= (m,-m,)v ay and from conservation of kinetic energy, eee or, mw = (m, +m)? Q) From £q, (1) and (2), myuw= oe Sur Tem -—-Cilaneeeenne or, ms -3m,m,= 0 Hence me z as =O oe (>) When they fly apart symmetrically relative to the initial motion direction with the angle of divergence 0= 60°, From conservation of momentum, along horizontal and vertical direction, my uy = m, v, cos (8/2) + ma v2 cos (8/2) @) and m, v, sin (8/2) = ma v2 sin (0/2) on, mv" mH V2 Q) Now, from conservation of kinetic energy, 1 1 1 FMM tm rm yt amy @) From (1) and (2), mv amy uy = 005 (8/2) |m, vy + m| = 2m, vs 008 (0/2) 1.170 So, 4, = 2 v, cos (8/2) (4) From (2), (3), and (4) mm me 2 my or, 4.008? (0/2) = ooo Ae 4m, cos” (8/2) vi-= my vis my, or, = 400s” 2 271 m. and putting the value of 8, we get, fe =2 If (4,,Vy) are the instantaneous velocity components of the incident ball and (%%, Mey) are the velocity components of the struck ball at the same moment, then since there are no external impulsive forces (i.e. other than the mutual interaction of the balls) We have usina= vy, vay "0 Mu COSA = mV, +m Vo, The impulsive force of mutual interaction satisfies da F d gad A> a Mw) (F is along the x axis as the balls are smooth. Thus Y component of momentum is not transferred.) Since loss of KE. is stored as deformation energy D, we have _ a. D zm wv Bry ivy 2 = dm? costa - Lmv,2- dmv,? 2 22 2 = BL nteteostar- Pry (mucosa my,)? ] = = [ 2mucosav,, - 2m?v,,2 ] = m(v,tcosa - v,,”) 2 = | weos’a. _ ( cose. } 4 2 fal ‘We see that D is maximum when 4 cosa, yn 2, a 7 ce 4 Pox 1 20 1b Then 0 qosae f smu 2 On substiuting a= 45° x & 1.171 From the conservation of linear momentum of the shell just before and after its fragmentation 1.172 Wests @ where ¥;, ¥7 and ¥3 are the velocities of its fragments. From the energy conservation 3yv?=v3 +343 (2) Now Vor Vig = V7~ Vem vE-V" @) where ve =vs velocity of the C.M. of the fragments the velocity of the shell. Obviously in the C.M. frame the linear momentum of a system is equal to zero, so a Se oe ime 0 @ Using (3) and (4) in (2), we get Sv = TEP + TIP + OUP = 37 4 293 + 292 4200-03 or, 26? + 25, F, cosO + 273+3(1-n)=0 6) If we have had used ¥5=-01~ 05, then Eq. 5 were contain 7; instead of ¥, and so on. The problem being symmetrical we can look for the maximum of any one. Obviously it will be the same for each. For ¥;to be real in Eq. (5) 453 cos@ = 8(273 +3 (1-1) ) or 6(y - 1) = (4 - cos0)F 3 So, hsv ae ot Fagan V2 (0-1) v = cos Hence ¥24aan) = [7% Pala, = +V2(n-1) vev(tsV2q-0 =1km/s Thus owing to the symmetry Yuan) ¥2 (ax) * Vay = ¥ (1 + V2(n = 1)) = 1 km/s Since, the collision is head on, the particle 1 will continue moving along the same line as before the collision, but there will be a change in the magnitude of it’s velocity vector. Let it starts moving with velocity v, and particle 2 with v, after collision, then from the conservation of momentum mu= mv, + mv, Of, Ua Vy + V2 ql) And from the condition, given, 1 iL 1 = mvi += mv3 pme-fmiramt) eg =1- we or, Vite ne Q) From (1) and (2), vi+(u-v)= -n)w or, view —2uv, += (1-1) a? 90 1.173 1.174 or, Wwi-2v,u+nue= 0 2 z a ne oe eo = }[usViF-2n? |= dua evi=m) Positive sign gives the velocity of the 2nd particle which lies ahead. The negative sign is correct for vy. So, v= fu (1-vI-2 ) = 5 m/s will continue moving in the same direction. Note that v,= 0 if n= 0 as it must. Since, no external impulsive force is effective on the system “M + m’”, its total momentum along any direction will remain conserved. So from p, = const. u mum Mv, cos® ot, vy= py Q@ and from py = const . Mea : vp = Mv, sin ot, v2= 7 vsinO= wtanO, [using (1)] Final kinetic energy of the system dot Tp= zis My And initial kinetic energy of the system= Smut T,-T, So, % change = at x 100 1 wl =m tan? 0+ = Lym =z mu 2 2 | ee ee Line am 4 Laing ey? sec20- dy? _ an 0+ FpU sec? O- > mn Le = [imo + Ff sec'0 1) x 100 and putting the values of @ and = , we get % of change in kinetic energy= - 40 % 7 , (a) Let the particles m, and m, move with velocities ¥; and ¥; respectively. On the basis Pi 1 m 1 2 y- of solution of problem 1.147 (b) : o Be yy = [7-3 1175 1.176 91 As WL ~ m, So, B= Vitae? where p= 2 im, +7, (b) Again from 1.147 (b) ay 1 2 T= Swi 5h [a-%| So, 7 Lucted From conservation of momentum P= Pi +P mer? ’ He ' so (i-Pr') = Pi-2p, py c0s8, + py!? = py’? From conservation of energy fe ae 2m, 2m, Eliminating p,' we get vA m , ae vont B) anna oni 2) This quadratic equation for p,' has a real solution in terms of p, and cos 0, only if 2" 4.c0s%0, mf =| > m B mi or sin? Os + my ns oot & 7 3 m or sin,<+ 2 or sind,z~—? m m ip! This clearly implies (since only + sign makes sense) that 7 in® my sin ® ax ™ 7 From the symmetry of the problem, the velocity of the disc A will be directed either in the initial direction or opposite to it just after the impact. Let the velocity of the disc A after the collision be v’ and be directed towards right after the collision. It is also clear from the symmetry of problem that the discs B and C have equal speed (say v”) in the directions, shown. From the condition of the problem, Fs, sinQ« Va-we /2 q) For the three discs, system, from the conservation of linear momentum in the symmetry direction (towards right) mv= 2mv"sinO+my' of, v= 2v"sinO+v! Q) 92 1177 1.178 From the definition of the coefficeint of restitution, we have for the discs A and B (or C) But e= 1, for perfectly elastic collision, So, vsin 0 = v"—v' sinO @) From (2) and (3), A ve Y= 2sin? 9) \ (1 +2 sin? 6) a - “or {using (1)} Hence we have, va Ver =2) 6-H Therefore, the disc A will recoil if 1 < V2 and stop if y= V2. Note : One can write the equations of momentum conservation along the direction per- pendicular to the initial direction of disc A and the consevation of kinetic energy instead of the equation of restitution. (a) Let a molecule comes with velocity vj to strike another stationary molecule and just after collision their velocities become V”, and ¥”, respectively. As the mass of the each molecule is same, conservation of linear momentum and conservation of kinetic energy for the system (both molecules) respectively gives : Deets and vie vit'S From the property of vector addition it is obvious from the obtained Eqs. that Vy LV) or ¥7,-5= 0 (b) Due to the loss of kinetic energy in inelastic collision v; > v'? + v'3 so, ¥°,+¥")>0 and therefore angle of divergence < 90°. Suppose that at time 4 the rocket has the mass m and the velocity v”, relative to the reference frame, employed. Now consider the inertial frame moving with the velocity that the rocket has at the given moment. In this reference frame, the momentum increament that the rocket & ejected gas system acquires during time dt is, ee oe dp= mdv+dtit= F dt or, mie F- wii 1.179 1.180 1.181 93 According to the question, F= Oandp = — dm/dt so the equation for this system becomes, mit, ini» dt” dt As dvt}i” so, mdv= -udm. Integrating within the limits : : s 1fa-- de Ye in ™ u mn 0 mo Thus, v= uin— m mo As dv't{ i so in vector form v= -#In > According to the question, F (external force) = 0 So, nt. he , a a" As at iy so, in scalar form, mdv= -udm wdt dm or, “Ss u m Integrating within the limits for m (1) wt dn v ee ee u m u my my Hence, m= me“) As F= 0, from the equation of dynamics of a body with variable mass; av” adm wi o ma eG dv= iv Now dv"} jii’and since i"L, ¥; we must have | dv"| = a (because vp is constant) where do. is the angle by which the spaceship turns in time df. udm So, -u Meyda o daw -4+™ m Vo m . _ fens (3) % 94 1.182 1.183 1.184 We have 2 = =p on dm= - pdt Integrating i dm= - ™ As = 0 so, from the equation of variable mass system : dv > dv iy > (my wt) Gem Foot, SP = We F/(mg- wt) ' or, far F/{ —4— (mp - 2) 0 0 Hence oe Ey, (2 Bo | my BE Let the car be moving in a reference frame to which the hopper is fixed and at any instant of time, let its mass be m and velocity 9" Then from the general equation, for variable mass system. a Fre an We write the equation, for our system as, dv” > -sdm — morn F-0S as, = -9 (1) So 4 Gi =F BR and v= ton integration. m But m= m+ pt od so, v= ———— mg (: + m| mo = > Thus the sought acceleration, w= 2. —__, my|1+ x mo Let the length of the chain inside the smooth horizontal tube at an arbitrary instant is x. From the equation, —» => —»dm mw= F+u dt 1.185 95 —> ia? > as i= 0, F tt w, for the chain inside the tube Axwe T where K= 7 Oy Similarly for the overhanging part, AU Tr w= 0 cee Thus mw= F Y T or Ahw= hhg-T 2) h From (1) and (2), hg. AQ +h) w= RAE oF, w+hvZ= he or, GeMye ips [As the length of the chain inside the tube decreases with time, ds = - dx] or, vdve ash Integrating, foes “en f ah (eh) of, am gh In (i) or v= V2¢h In (i) Force moment relative to point O ; > dM o N= —~= 2bt dt Let the angle between M and N, a= 45° att = toe fn fh ee @+b%)- di) on 2 HON, Ce VE" [MIN] Va + 071 2b wie big Vat+b te 2p Var ty > bt2 a/@ So, 2b? ibm ak +b? tg oF, tom V5 (@s fo cannot be negative) It is also obvious from the figure that the angle a is equal to 45° at the moment fo, when a= big, ie. fy = Va7b and N= avin 96 1.186 1,187 1.188 MW)= 7p (e382 xm Ve +B) 1x 2 = mvg gt? sin ct ack y+ 5 muy gt * sin +a)@) = 4 mvg gt? 008.0 (-#): mvp 8 t? cos 2 Thus angular momentum at maximum height « vosina teat t= FeO Thus M (1) = s.- 3 ‘t\ {mo} 2 . 2 1 (5)- (Get st aconae a7hg-n? Alternate : TIIVVIVTITI7?X. > => ; > ; M()=0 50, M()= f Nar= f (Femz) 0 0 ' -f (rege SCE eae f [(@r+2e") xm] (ema) 3 (a) The disc experiences gravity, the force of reaction of the horizontal surface, and the force R of reaction of the wall at the moment of the impact against it. The first two forces counter-balance each other, leaving only the g force R. It’s moment relative to any point of A the line along which the vector R acts or along normal to the wall is equal to zero and therefore “ > the angular momentum of the disc relative to 9 any of these points does not change in the given process. (b) During the course of collision with wall the position of disc is same and is equal to FZ, Obviously the increment in linear momentum of the ball Ap™= 2mv cos a” Here, AM = 72, x Ap™= 2 mv cos an and directed normally emerging from the plane of figure Thus |AM|= 2mvlcosa (a) The ball is under the influence of forces T and m g’at all the moments of time, while QS moving along a horizontal circle. Obviously the vertical component of T balance m g’and 1.189 1.190 97 So the net moment of these two about any point becoems zero. The horizontal component of T, which provides the centripetal acceleration to ball is already directed toward the centre (C) of the horizontal circle, thus its moment about the point C equals zero at all the moments of time. Hence the net moment of the force acting on the ball about point C equals zero and that’s why the angular mommetum of the ball is conserved about the horizontal circle. (b) Let o be the angle which the thread forms with the vertical. Now from equation of particle dynamics : Tcosa= mg and Tsina = mw’ Isina 1 . or @ As |M | is constant in magnitude so from figure. |AM| = 2M cos a where M~ |M;|= |M,| = |xmv |= my I (as iio LV) Hence on solving cos 0. = Thus|AM| = 2 mv 1cos a= 2 mw I? sin a cos 2mgl /, e = a &] (using 1). , the reference point O moves in hoizontal direction During the free fall time t= v = (say towards right) by the distance V7. In the translating frame as M (0) =0, so a . ~> AM=M;=7r . iW) = (-Vuithj )xm[gtj-Vi] a = -mV girs mVh(+é) J (9) 2h) = . : -mve(F|Femvaed) = -mVhE Hence |AM|= mVA The Coriolis force is.(2m 0" x @). Here @ is along the z-axis (vertical). The moving disc is moving with velocity vy which is constant. The motion is along the x-axis say. Then the Coriolis force is along y-axis and has the magnitude 2m vo. At time ¢, the distance of the centre of moving disc from O is vot (along x-axis). Thus the torque N due to the coriolis force is N= 2mvg eve along the z-axis. 1.191 1.192 Hence equating this to ae a 2m vot or M= mvoct? + constant. The constant is irrelevant and may be put equal to zero if the disc is originally set in motion from the point O. This discussion is approximate. The Coriolis force will cause the disc to swerve from straight line motion and thus cause deviation from the above formula which will be substantial for large t If += radial velocity of the particle then the total energy of the particle at any instant is 2 where the second term is the kinetic energy of angular motion about the centre O. Then kmi?s Ms we (1) the extreme values of r are determined by 7 = 0 and solving the resulting quadratic equation HepB +n 9 we get From this we see that £=Kn +A) (2) where r, is the minimum distance from O and r, is the maximum distance. Then i Tyo sed kd + 73) 2k? Hence, m= i] ‘Note : Eq. (1) can be derived from the standard expression for kinetic energy and angular momentum in plane poler coordinates : 1.21 29 T=zmi?+5mre M = angular momentum = mr’6 The swinging sphere experiences two forces : The gravitational force and the tension of the thread. Now, it is clear from the condition, given in the problem, that the moment of these forces about the vertical axis, passing through the point of suspension N, = 0. Con- sequently, the angular momentum M, of the sphere relative to the given axis (z) is constant. Thus myg (I sin @) = mv 1 Q) where m is the mass of the sphere and v is it s velocity in the position, when the thread forms an angle 5 with the vertical. Mechanical energy is also conserved, as the sphere is 1193 1.194 1.195 99 under the influence if only one other force, i.e. tension, which does not perform any work, as it is always perpendicular to the velocity. So, imi +mglcos 0= im? (2) From (1) and (2), we get, Vg = V2gI7cos © Forces, acting on the mass m are shown in the figure. As N= mg’ the net torque of these two forces about any fixed point must be equal to zero. Tension T, acting on the mass m is a central force, which is always directed towards the centre O. Hence the moment of force T is also zero about the point O and therefore the angular momentum of the particle m is conserved about O. Let, the angular velocity of the particle be @, when the separation between hole and particle m is r, then from the conservation of momentum about the point O, : m (7p) ro = m(wr)r, or or a r Now, from the second law of motion for m, T= F= mor Hence the sought tension; mogrér moard Foe —_—= ne re F On the given system the weight of the body mis the only force whose moment is effective about the axis of pulley. Let us take the sense of @ of the pulley at an arbitrary instant as the positive sense of axis of rotation (z-axis) As M,(0)= 0, s0, AM, = M, (= f N,de ' So, M,@= f mgRdt= mg Rt Oo Let the point of contact of sphere at initial moment (t= 0) be at O. At an arbitrary moment, the forces acting on the sphere are shown in the figure. We have normal reaction N, = mg sin a. and both pass through same line and the force of static friction passes through the point O, thus the moment about point O becomes zero. Hence mg sin a: is the only force which has effective torque about point O, and is given by |N |= mgRsina normally emerging from the plane of figure. As M(t= 0)= 0, so, A= (= f Nar Hence, M(t)= Nt= mgRsinot 100 1.196 1.197 1.198 Let position vectors of the particles of the system be O and O' respectively. Then we have, nd 7; with respect to the points i= +79 () where 76 is the radius vector of O’ with respect to O. Now, the angular momentum of the system relative to the point O can be written as follows; M- > (7) > (F =) > (ez?) [using (1)] or, M-M 4 (oP), where, p= 7 Q From (2), if the total linear momentum of the system, p= 0, then its angular momen- tum does not depend on the choice of the point O. Note that in the C.M. frame, the system of particles, as a whole is at rest. On the basis of solution of problem 1.196, we have concluded that; “in the C.M. frame, the angular momentum of system of particles is independent of the choice of the point, relative to which it is determined” and in accordance with the problem, this is denoted by M. We denote the angular momentum of the system of particles, relative to the point O, by M,. Since the internal and proper angular momentum Mf, in the C.M. frame, does not depend on the choice of the point O’, this point may be taken coincident with the point O of the K-frame, at a given moment of time. Then at that moment, the radius vectors of all the particles, in both reference frames, are equal (7; = 7;) and the velocities are related by the equation, - - Wee vier, @) where 7? is the velocity of C.M. frame, relative to the K-frame. Consequently, we may write, x7) +3 m (We) on, Ma Mm (7x72), as) miv= mit, where m= Ym. or, M= Ws (72x miv7)= M+ (7x3) From conservation of linear momentum along the direction of incident ball for the system consists with colliding ball and phhere myg= mv + Oy, Q) where v’ and v, are the velocities of ball and sphere 1 respectively after collision. (Remember that the collision is head on). As the collision is perfectly elastic, from the definition of co-efficeint of restitution, vny 22 OE of v= y= -% (2 1.199 101 Solving (1) and (2), we get, aNo | : Vo y= directed towards right. @->--- m mp A ; In the CM. frame of spheres 1 and 2 (Fig,) Pi = ~ Bp and [P;1= [22] = wl - FI Also, Fic = -Fac, thus M = 2[73-xpi] Cc 1 m/2 4% » oe sy 4 % (where 7 is the unit vector in the sense of Fx Py ) mp As Fic L Diy wot mv! Hence M= In the CM. frame of the system (both the discs + spring), the linear momentum of the discs are related by the relation, p;= - p> at all the moments of time. where, By= By= B= BV pa And the total kinetic energy of the system, T= de v2, [See solution of 1.147 (b)] Bearing in mind that at the moment of maximum deformation of the spring, the projection of vz, along the length of the spring becomes zero, i.e. V,.1(.) = 0- The conservation of mechanical energy of the considered system in the C.M. frame gives. 1(m 1 1 (m' a 3% 3e+5 (5) M0 ® Now from the conservation of angular momentum of the system about the C.M., Ble} -1 or, van” Toray a(t +f] a a (2) Using (2) in (1), smals-(-§) nx? 1 ve x 2) 2 or, Peleg ware =K myx : or, i = «2x, [neglecting x” / 13] mM% As x= 0, thus x= —— Kip 102 1.4 UNIVERSAL GRAVITATION 1.200 We have Mee ie pe Thus oe eee roym/v? ym, (Here m, is the mass of the Sun.) Qnym, 2m x 667x107" x 197 x 10% So T= St = SxOON eee y (349 x 107) (The answer is incorrectly written in terms of the planetary mass M) = 1-94. 107 sec = 225 days. 1.201 For any planet M. MRote oe or a yf LM R R? So, Te 2a. 2nR*?/vVym, 372 T, ‘Ry a) Thus oe | @ z- (r s - 23 23 0 Ron lol Te)? = (12)? = 5-24. te wa 2m, (pV ) Vim pts and Ry (- oo _ mF en? 2 So v} 7 where T= 12 years. m,= mass of ths Sun. Putting the values we get V, = 12-97 km/s 2 . Vy (2R1 mM, 2x Aestemion = Fe (257%) (8) 4a - (7) (ym, )? = 215% 10 km/s? 103 1.202 Semi-major axis= (r + R)/2 r+R 2 It is sufficient to consider the motion be along a circle of semi-major axis for T does not depend on eccentricity. r+R ie 2n\ 2 "Ke xV(r+R)/2ym, (again m, is the mass of the Sun) Hence T= 1.203 We can think of the body as moving in a very elongated orbit of maximum distance R and minimum distance 0 so semi major axis = R/2. Hence if t is the time of fall then 2 - 2x) _ (R22 7 (#) (# ) or = 77/32 or t= T/4V2 = 365 / 4v2 = 645 days. 1.204 T= 22R°?/Vym, If the distances are scaled down, R°” decreases by a factor 1°”and so does m, . Hence T does not change. mm, 1.205 The double star can be replaced by a single star of mass m, +m, moving about the centre of mass subjected to the force ym, m,/ 1°. Then 2x? 2 he a ade So Pr nV v3 of, re (zs) (My? = WyM (7/2? 2n 1.206 (a) The gravitational potential due to m, at the point of location of m, : oe ym, ym, a= f Sars f Mae > , r ymm So, Up, = my Vy = - r Similarly Cee r 104 1.207 ee mt ma ymm, o—__,—___#-—----| }—->x Uys Uy = Us --— ne E ax () Choose the location of the point mass as the origin. Then the potential extergy dU of an element of mass dM = Mae of the rod in the field of the point mass is M1 dU =-ym yee where x is the distance between the element and the point. (Note that the rod and the point mass are on a straight line.) If then a is the distance of the nearer end of the rod from the point mass. ---------- 2 | k§—____—_———__ x ———__>™ ae ay! mM [ de M L u--f ry Ss ~-with( 1+) The force of interaction is au Fe- aa mM ak a aver 17 a? a(a+) 1 z) ° ymM a se oe a Minus sign means attraction. As the planet is under central force (gravitational interaction), its angular monientum is conserved about the Sun (which is situated at one of the focii of the ellipse) an So, my,r= mVzr, OF, 4- (qa) 1 From the conservation of mechanical energy of the system (Sun + planet), ym 1 ym,m 1 2 Ste - 7 +ymv, ym 1 a _ (tm) 1 . or, a Soi tale 7 +3% [Using (1)] Thus, vy= V2ym,7,1% +h) Q) Hence M= myyry= mV2y m, ry Fq/ (ry +7) 1.208 1.209 105 From the previous problem, if r, , r, are the maximum and minimum distances from the sun to the planet and v, , v2 are the corresponding velocities, then, say, E=b aw? -™™ 2 7 _ymm, ry ym, ym, ee Myth 2 Me ntl, 2q (sing Ea. (2) of 1.207] where 2a = major axis = r, +r, The same result can also be obtained directly by writing an equation analogous to Eq (1) of problem 1.191. Eady? “2! 2m or (Here M is angular momentum of the planet and m is its mass). For extreme position r= 0 and we get the quadratic EP +4mmy - we =0 The sum of the two roots of this ae are , nytry=- a2 Thus nee constant From the conservtion of angular momentum about the Sun. mvp roSina= mv, r,= mVy%y Of, v7, * Vy T= Yor, sina oy From conservation of mechanical energy, 1 ymzm 1 ym,m zm zi rm vest ym, or, > 2% ar . So, v7 sin eB 2) r[12V1-@-wnswa | 1s ym, |r 7m ees 7 @-9) 1% [ ym, where 1 = ¥O7y/ym,, (1m, is the mass of the Sun). 106 1.210 1.201 At the minimum separation with the Sun, the cosmic body’s velocity is perpendicular to its position vector relative to the Sun. If 7,,, be the sought minimum distance, from con- servation of angular momentum about the Sun (C). Vol Mv l= MVF pin OT, V — q@) ‘min From conservation of mechanical energy of the system (sun + cosmic body), 1 ymem 1 Ce eee 2 Tain 2 nae 2 y my So, es S— (using 1) 2 Tain Wein or, v9 roi, + 2Y M,raig VEL? = 0 = 2y m,2 Vay? m? + ave v2 ym eV? me + vee od ee 2ve vg Hence, taking positive root Tein * (ym, 13) [Vie a7, - 1] Suppose that the sphere has a radius equal to a. We may imagine that the sphere is made up of concentric thin spherical shells (layers) with radii ranging from 0 to a, and each spherical layer is made up of elementry bands (rings). Let us first calculate potential due to an elementry band of a spherjcal layer at the point of location of the point mass m (say point P) (Fig.). As all the points of the band are located at the distance J from the point P, so, _. -13M (where mass of the band) @ am-( a? | (28nd) (048) Ana : (<")sino ao oy And [?= a?+r?-2arcos® (3) Differentiating Eq. (3), we get Idl = ar sin dO 4 Hence using above equations aon -(Sir ja © 1.212 1.213 107 Now integrating this Eq. over the whole spherical layer aM donfage tat f So do- -1t © Equation (6) demonstrates that the potential produced by a thin uniform spherical layer outside the layer is such as if the whole mass of the layer were concentrated at it’s centre; Hence the potential due to the sphere at point P; wm p= fdg~ -tfame-™ M This expression is similar to that of Eq. (6) Hence thte sought potential energy of gravitational interaction of the particle m and the sphere, U= mp= - im (©) Using the Eq., G,= -22 G,= "7 (using Eq. 7) So G= MF ang Fe mG => 8) (The problem has already a clear hint in the answer Sheet of the problem book). Here we adopt a different method. Let m be the mass of the spherical layer, wich is imagined to be made up of rings. Ata point inside the spherical layer at distance r from the centre, the gravitational potential due to a ring element of radius a equals, dp= - oa dl (sce Eq. (5) of solution of 1.211) So, p= fdp= - fa--™ Q) Hence G,- -2a0. Hence gravitational field strength as well as field force becomes zero, inside a thin sphereical layer. One can imagine that the uniform hemisphere is made up of thin hemispherical layers of radii ranging from 0 to R. Let us consider such a layer (Fig.). Potential at point O, due to this layer is, 108 1.214 2 dg= 4m. 3M iy, where dm = —M __| Sr) ay r R (2/3)nR>\ 2 (This is because all points of each hemispherical shell are equidistant from O.) R Hens, = fadgm 20 frie « Set Se Hence, the work done by the gravitational field force on the particle of mass m, to remove it to infinity is given by the formula A= mg, since @ = 0 at infinity. Hence the sought work, 3ymM Ap+e™ one (The work done by the external agent is - A.) In the solution of problem 1.211, we have obtained @ and G due to a uniform shpere, at a distance r from it’s centre outside it. We have from Eqs. (7) and (8) of 1.211, Mi yM> on -T* and G= -7 (A) Accordance with the Eq. (1) of the solution of 1.212, potential due to a spherical shell of radius a, at any point, inside it becomes ie pL pm 1 = Const. and G,= -5P= 0 @) For a point (say P) which lies inside the uniform solid sphere, the potential @ at that point may be represented as a sum. Oe ier where @, is the potential of a solid sphere having radius r and 9, is the potential of the layer of radii r and R. In accordance with equation (A) (eas ee 1 ee oan ) pe The potential ¢, produced by the layer (thick shell) is the same at all points inside it. The potential ~, is easiest to calculate, for the point positioned at the layer’s centre. Using Eq, (B) neon f ober -r) where dM = 4nr?dr= - Pdr i (4/3) xR? is the mass of a thin layer between _ radii r and r+ dr. ~ Preside = 01+ P2™ (3% n)>-3] © 1.215 109 From the Eq. Ge =z yMr G,= R eo ie or G R r YZ "er M . ns (where p = 7, is the density of the sphere) ©) zaR 3 The plots @ (r) and G (r) fora uniform sphere of radius R are shown in figure of answersheet. Alternate : Like Gauss’s theorem of electrostatics, one can derive Gauss’s theorem for ooo . gravitation in the form § G-dS= -4nY Minciosea - For calculation of G at a point inside the sphere at a distance r from its centre, let us consider a Gaussian surface of radius r, Then, M M G,4nP= -4ny By os, G= -Wy R? . R So, on f gare f Mh rare f ha r r Ae R Integrating and summing up, we get, iM (2 : zs) tines, Bo IME -v$apr up ZA) "oR Re ‘And from Gauss’s theorem for outside it : YM 2 G,4nr?= —AnyM ot G,= —Tp Thus ot f G,ar= 7 Treating the cavity as negative mass of density - p in a uniform sphere density + p and using the superposition principle, the sought field strength is : 3-342, or G= ~4ny pF, + -Fyx(-—) (where 77 and 7” are the position vectors of an orbitrary point P inside the cavity with respect to centre of sphere and cavity respectively.) n-r Thus = -Fav0(7 )-3arel 110 1.216 1.217 ‘We partition the solid sphere into thin spherical layers and consider a layer of thickness dr lying at a distance r from the centre of the ball. Each spherical layer presses on the layers within it. The considered layer is attracted to the part of the sphere lying within it (the outer part does not act on the layer). Hence for the considered layer dp4nr*= dF cea) ie) aartarn r (where p is the mean density of sphere) or dP4nxr?’= o, dp= fay ptrdr R Thus pf ap- By (R21) 4 (The pressuresmust vanish at r = R.) 3 or, Ling) 1- (?7R?)) y M7 xR‘, Putting p= M/(4/3) xR? Putting r= 0, we have the pressure at sphere’s centre, and treating it as the Earth where mean density is equal to p = 5-5 x 10° kg/m? and R= 64x 107 km we have, p= 1-73x10"Pa or 1-72 x 10° atms. (@) Since the potential at each point of a spherical surface (shell) is constant and is equal to p= - ye [as we have in Eq. (1) of solution of problem 1.212) ‘We obtain in accordance with Aes — u-4 jSama~ 5 dof am . if 7%) me UE. 2\'R 2R (The factor Sis needed otherwise contribution of different mass elements is counted twice.) (b) In this case the potential inside the sphere depends only on r (see Eq. (C) of the solution of problem 1.214) 2 3R? ) Here dm is the mass of an elementry spherical layer confined between the radii rand r+dr: dm= (4nrdrp)= (ae \r ar Mi “Stale ls After integrating, we get yn U=- R wie 1218 Leto= ‘Y “AE = circular frequency of the satellite in the outer orbit, : YMe (r-Ar? = = circular frequency of the satellite in the inner orbit. So, relative angular velocity = wy + w where - sign is to be taken when the satellites are moving in the same sense and + sign if they are moving in opposite sense. Hence, time between closest approaches 2x 2x 1 { 45 days a a = = 0-80 hour = 2) M+ oO VyM, |r? 2Ar es where 6 is 0 in the first case and 2 in the second case. yM _ 667x107! x 5:96 x 10% = = 98 m/s” Re (63710 : 2 2 2 2x 2x22 | 2 @,= 0 R= (F ) R (saxaet037 ) 637 x 10° = 0-034 m/s’ _ IMs | 667x107 x 1.97 x 10° os = 59x 10% m/s? Re (149-50 x 10° x 10°) and Then @,: @,: @; = 1: 0-0034 : 0:0006 1.220 Let h be the sought height in the first case. so ae 100° (R+h) 112 1.221 1.222 1.223 -2 or bd 1+> 7 100" R From the statement of the problem, it is obvious that in this case h< (V2 -1)= gR(V2-1) This ‘kick’ in velocity must be given along the direction of motion of the satellite in its orbit. Let r be the sought distance, then 9M Me 2 2 a 2m (mR - GoR=rye pe Rr) or Vira (nR-r) oc pe en 3.8 x 10° kon. wel Between the earth and the moon, the potential energy of the spaceship will have a maximum at the point where the attractions of the earth and the moon balance each other. This maximum PE. is approximately zero. We can also neglect the contribution of either body to the p.E. of the spaceship sufficiently near the other body. Then the minimum energy that must be imparted to the spaceship to cross the maximum of the PE. is clearly (using E to denote the earth) 1.233 117 yMgn Rk With this energy the spaceship will cross over the hump in the PE. and coast down the hill of p.E. towards the moon and crashland on it. What the problem seeks is the minimum energy reguired for softlanding. That reguies the use of rockets to loving about the braking of the spaceship and since the kinetic energy of the gases ejected from the rocket will always be positive, the total energy required for softlanding is greater than that required for crashlanding. To calculate this energy we assume that the rockets are used fairly close to the moon when the spaceship has nealy attained its terminal velocity on the moon 4 / 2M, R where M, is the mass of the moon and Ry is its radius. In general 0 dE = vdp and since the speed of the ejected gases is not Jess than the speed of the rocket, and momentum transfered to the ejected gases must equal the momentum of the spaceship the energy E of the gass ejected is not less than the kinetic energy of spaceship yMgn Ry Addding the two we get the minimum work done on the ejected gases to bring about the softlanding. A (Mg M, me WLR, * Ry On substitution we get 1:3 x 10° Wd. Assume first that the attraction of the earth can be neglected. Then the minimum velocity, that must be imparted to the body to escape from the Sun’s pull, is, as in 1-230, equal to (V2 -1)¥, where v2 = yM,/r,r= radius of the earth’s orbit, M,= mass of the Sun, In the actual case near the earth, the pull of the Sun is small and does not change much over distances, which are several times the radius of the Earth. The velocity v in question is that which overcomes the earth’s pull with sufficient velocity to escape the Sun’s pull. Thus 1. Me 1 2 2 zm gm (V2 1) vy where R = radius of the carth, M, = mass of the earth. Writing v? = yM, /R, we get vy= V2v2+(V2-1)' v2 = 166 km/s 118 1.5 DYNAMICS OF A SOLID BODY 1.234 1.235 1.236 1.237 Since, motion of the rod is purely translational, net torque about the C.M. of the rod should be equal to zero. Lop(t fe Thus P57 Fi(z-4) Ria @ For the translational motion of rod. en F, mw, 2 27H = mw, one Fy 2) From (1) and (2) a me a 26s Sought moment N= 7% F= (ai+bj)x(Ai+Bj) = aBk+Ab(-K)= (aB-Ab)k” N_ aB-Ab and arm of the force Les SSS Fo VA"+B™ Relative to point ¢ O, the net moment of force : Na rx F,+7xFy= (aixAj)+(Bjx Bi) = abk+AB(-k)= (ab-AB)E- (@) Resultant of the external force ee Fe F,+F,= Aj+Bi Q2) As N-F= 0 (as NLF) so the sought arm / of the force F_ ab-AB VA? + BP For coplanar forces, about any point in the same plane, 7; 7x Fem rx Fey I= N/F= (where F, = yr = resultant force) of, Nig, = 7% Fee Nnet Thus length of the arm, != =" Fre Here obviously [F,., |= 2F and it is directed toward right along AC. Take the origin at C. Then about C, Ne (vVFaF+ wF- V2 aF | directed normally into the plane of figure. (Here a = side of the square.) Thus N= Fe directed into the plane of the figure. F(a/v2 a a Ss Hence I= oF" OVE sin 45' Thus the point of application of force is at the = point of the side BC. 1238 1.239 119 (a) Consider a strip of length dx at a perpendicular distance x from the axis about which we have to find the moment of inertia of the rod. The elemental mass of the rod equals m dm= 1 a& Moment of inertia of this element about the axis l= dmx? = 7 dex? Thus, moment of inertia of the rod, as a whole about the given axis (b) Let us imagine the plane of plate as xy plane taking the origin at the intersection point of the sides of the plate (Fig.). 0 Obviously I= f dmy? ee -f (wa)>? Oo _ ma? 3 i Smmilarly i= me Hence from perpendicular axis theorem Myr p2 L=1,+1,= 3 (4 +b ) which is the sought , moment of inertia. (a) Consider an elementry disc of thickness dx. Moment of inertia of this element about the z-axis, passing through its C.M. A : at,» ODE 9 sae co 4 where p = density of the material of the plate + and S = area of cross section of the plate. Thus the sought moment of inertia b b 2 2 _ PSR f wf 12 PD J x= “> pSb - FPbR*(asS= xR?) 120 1.240 putting all the vallues we get, I, = 2: gm-m? (b) Consider an element disc of radius r and thickness dx at a distance x from the point O. Then r= x tana. and volume of the disc = we? tan? ade Hence, its mass dm= xx? tana. de-p (where Moment of inertia of this element, about the axis OA, 3 i { \ p= density of the cone= m/yxR*h) — Saat) WS 2 r d= dm x? tana = (xx? tan7a dr) 7 . AP xt tan ‘ade h ‘Thus the sought moment of inertia = 2 tan‘ a f ah de 0 Hence 3m R? (unin oa 10” [Puting P= =, (a) Let us consider a lamina of an arbitrary shape and indicate by 1,2 and 3, three axes coinciding with x, y and z— axes and the plane of lamina as x ~y plane. Now, moment of inertia of a point mass about x- axis, df, = dmy” Thus moment of inertia of the lamina about this axis, 1, =f dmy? Similarly; J, = f ‘dmx? and I, = f dmr? = fidm(x?+y?) as r= Vx7+y™ Thus, T= 1,41, of, I= 1+, (b) Let us take the plane of the disc as x -y plane and origin to the centre of the disc ig.) From the symmetry [,= 1,. Let us consider a ring element of radius r and thickness dr, then the moment of inertia of the ring element about the y - axis. 1.241 121 d[,= dmr? = aya (omar)? Thus the moment of inertia of the disc about z— axis R 2m 3, mR? i= mr dre But we have I= 1,+1,= 4, I, mR? Thus heyeG For simplicity let us use a mathematical trick. We consider the portion of the given disc as the superposition of two. complete discs (without holes), one of positive density and radius R and other of negative density but of same magnitude and radius R/2. As (area) a (mass), the respective masses of the considered discs are (4m /3) and (-m/3) respectively, and these masses can be imagined to be situated at their respective centers (C.M). Let us take point O as origin and point x - axis towards right. Obviously the C.M. of the shaded position of given shape lies on the x - axis. Hence the C.M. (C) of the shaded portion is given by (-m/3)(-R/2)+(4m/3)0) pz a8 (=m/3)+4m/3 2G Thus C.M. of the shape is at a distance R/6 from point O toward x - axis Using parallel axis theorem and bearing in mind that the moment of inertia of a complete homogeneous disc of radius m, and radius rp equals 3 mgrg. The moment of inetia of the small disc of mass (-m/3) and radius R/2 about the axis passing through point C and perpendicular to the plane of the disc n.-2(-™)(R) ,(_-™)(2,8 2 ‘| = 3 ace | mia aa 2427 2 Similarly i 2(3)"" +(F)(8) 201, Re Sao Thus the sought moment of inertia, lem het le FemR?— SmR? = Fl mk? 122 1.242 Moment of inertia of the shaded portion, about the axis passing through it’s certre, Te 3(5**°°) 8? Now, if R = r + dr, the shaded portion becomes a shell, which is the required shape to calculate the moment of inertia. Now, [= Z-Sap|(r+dr)5—r5] Neglecting higher terms. : 2 (4nr? drp)r?= Zor? 1.243 (a) Net force which is effective on the system (cylinder M +body m) is the weight of the body m in a uniform gravitational field, which is a constant, Thus the initial acceleration of the body m is also constant. From the conservation of mechanical energy of the said system in the uniform field of gravity at time t= Ar :AT+AU= 0 1a AMR? or zm ta -mgAh=0 or, 5 (2m +MY v? ~ mg b= 0 [as v= oR at all times] But v? = 2w Ah Hence using it in Eq. (1), we get 4 (2m-+M) 20 Ah~mg Ah = 0 of w= mB (m+M) From the kinematical relationship, B = rR" aE Thus the sought angular velocity of the cylinder pee Sg se Sr, @(1)= Bre GM)R!” Ge M/am)R (b) Sought kinetic energy. 2 T(t)= mv? i ME? 5 (2m+M)R?0? 1.244 1.245 1.246 123 For equilibrium of the disc and axle 2T = mg or T= mg/2 As the disc unwinds, it has an angular acceleration B given by [B= 21r ot B= oh mar T The corresponding linear acceleration is 2 rB= w= T Since the disc remains stationary under the combined action of this acceleration and the acceleration (-w) of the bar which is transmitted to the axle, we must have ™g 2 mgr 7 Let the rod be deviated through an angle q’ from its initial position at an arbitrary instant of time, measured relative to the initial position in the positive direction. From the equation of the increment of the mechanical energy of the system. AT= Ag, we 1 or, zo 2. IN. dp ° 2 ME = f Fleospdp = Flsing oy SFsing Thus, o- i First of all, let us sketch free body diagram of each body. Since the cylinder is rotating and massive, the tension will be different in both the sections of threads. From Newton’s law in projection form for the bodies m, and m, and noting that w, = w,= w= BR, (as no thread slipping), we have (m, > m,) Nie m,g-T,= m,w= m, BR x and T)-m,g= m,w @) @ Now from the equation of rotational dynamics t of a solid about stationary axis of rotation. ie. q 2 N,= IB, for the cylinder. or, (T,-Ty)R= 1B = mR’ B/2 (2) : fe Similtancous solution of the above equations yields : 4, * wh (1m, - m,) g qe m,(m+4m,) (mem +2) 2) mg. mad. 124 1.247 1.248 As the systemt (m +m, +m, ) is under constant forces, the acceleration of body m, an ‘m, is constant. In addition to it the velocities and accelerations of bodies m, and m, at equal in magnitude (say v and w) because the length of the thread is constant. From the equation of increament of mechanical energy i.e. AT + AU = A,,, at time ¢ whe block m, is distance h below from initial position corresponding to t= 0, 1 1 (mR?) v? pm) Omg — km, gh a (as angular velocity = v/R for no slipping of thread.) But v? = 2wh So using it in (1), we get 2 (m, - km, ) 8 Ae er ied 2 m+2(m,+m,) Thus the work done by the friction force on m, 1 Ay, = — kanygh = tm (5 0?) fam, (my ~ kom, ) g? 1? : ” ~n+2.(m, +m) (using 2). In the problem, the rigid body is in translation equlibrium but there is an angular retardation. We first sketch the free body diagram of the cylinder. Obviously the friction forces, acting on the cylinder, are kinetic. From the condition of translational equlibrium for the cylinder, mg= N,+kN,; N= kN, mj mj Hence, N= reac Ny= ee For pure rotation ‘of the cylinder about its rotation axis, N, = JB, mR? or, - kN, R-kN,R= > B, KNg; kingR(1 +k) mR? eee a or, B,= - aan aah R + = KN, Now, from the kinematical equation, 7 = w) +28, AQ we have, wg (1+k?)R HCisEyg eae w= 0 1.249 125 Hence, the sought number of turns, So. we (1+ Re yR Qn” 8uk(1+k)g It is the moment of friction force which brings the disc to rest. The force of friction is applied to each section of the disc, and since these sections lie at different distances from the axis, the moments of the forces of friction differ from section to section. To find N,, where z is the axis of rotation of the disc let us partition the disc into thin rings (Fig.). The force of friction acting on the considered element dfr = k(2nrdro)g, (where o is the density of the disc) The moment of this force of friction is dN, = -rdfr= -2nkogr’dr Integrating with respect to r from zero to R, we get n= 2 2 : N,=-2nkog fr dr= -2nkogR®. z oO For the rotation of the disc about the stationary dr axis z, from the equation N, = 1B, Ey 1.250 Thus from the angular kinematical equation @,= Og, + Bt = __ dk _ FR 0 n+ aR |f orf ake According to the question, 122. Wa ot, I= Bm ~ kat Vo. a it Integrating, Vo = -57 + Van 2,2 or, o- Oo 4 45, QNoting that at = 0, = Wy) 2Ve@ k Let the flywheel stops at t= fo then from Eq. (1), f= Hence sought average angular velocity 2VOo k oe S(S eH say) a = 22 —______% vee 3 k fa 126 1.251 1.252 aM, Let us use the equation = = N, relative to the axis through O (Q For this purpose, let us find the angular momentum of the system M, about the given rotation axis and the corresponding torque N,. The angular momentum is mo 2 M, = Io + mvR = (Sema o [where I= SR? and v= wR (no cord slipping)] So, Be (Mr eme’)p, @ The downward pull of gravity on the overhanging part is the only external force, which exerts a torque about the z~axis, passing through O and is given by, N,= (F}-se ZoN Hence from the equation aN (‘Ss . oma’) = WxeR ong Thus, B= incaeomy?? Note : We may solve this problem using conservation of mechanical energy of the system’ (cylinder + thread) in the uniform field of gravity. (a) Let us indicate the forces acting on the sphere and their points of application. Choose positive direction of x and (rotation angle) along the incline in downward direction and in the sense of @ (for undirectional rotation) respectively. Now from equations of dynamics of rigid body ie. F,= mw, and N,,= I, B, we get : mg sin a f,= mw () and frR= 2mk 2B Q) But fr's kang cosa In addition, the absence of slipping provides the kinematical realtionship between the accelerations : w= BR @ The simultaneous solution of all the four equations yields : 2. 2 koosaz Fsina, or kz Ftana (b) Solving Eqs. (1) and (2) [of part (a)], we get : 1.253 1.254 127 oda wo= 7gsina. As the sphere starts at t= 0 along positive x axis, for pure rolling ¥.(O= wet= Zgsinart 6) Hence the sought kinetic energy Ee ta 2 . Tm Smet 7 5 mR? wr = so mvz (as w= v/R) 2 teh oe ee im (esas) qa me sin’ at (@) Let us indicate the forces and their points of application for the cylinder, Choosing the positive direction for x and @ as shown in the figure, we write the equation of motion of the cylinder axis and the equation of moments in the C.M. frame relative to that axis ie. from equation F, =, mw, and N,= I, B, 2 mg-2T = mw,; 2TR = ™—B As there is no slipping of thread on the cylinder w.= BR From these three equations = 78 = 28, ? rad/s? Tm Bm 13N, Bo FF 5x10? rad/s 2g (b) we have B= 58 So, we 2.0 or, in vector form w= 23° Pe Ft F-() ~ mB"(2a%)= mets Let us depict the forces and their points of application corresponding to the cylinder attached with the elevator. Newton’s second law for solid in vector form in the frame of elevator, gives : 2+ mg’t m(- ise) = mis” () The equation of moment in the C.M. frame relative to the cylinder axis i.e. from N,= IB, - mR? mR? w’ R= “BOR [as thread does not slip on the cylinder, w’ = BR ] 128 1.255 1.256 mw or, rT. mm As (I) Tt? so in vector form 7 mw 7. -% @ Solving Eqs. (1) and (2), # = 2g #) and sought force gic gee — =. F=2T= 3m (e-™)- Let us depict the forces and their points of application for the spool. Choosing the positive direction for x and @ as shown in the fig., we apply F,= mw,, and N.:= I, B, and get 0 mg sina -T= mw; Tr = IB “Notice that ifa point ofa solid in plane motion is connected with a thread, the projection of velocity vector of the solid’s point of contact along the length of the thread equals the velocity of the other end of the thread (if it is not slacked)” Thus in our problem, v,= vo but v= 0, hence point P is the instantaneous centre of rotation of zero velocity for the spool. Therefore v, = wr and subsequently w, = Br. .y Solving the equations simultaneously, we get ye cones =r = 1-6 m/s? mn 1+ ot 4 Let us sketch the force diagram for solid cylinder and apply Newton’s second law in projection form along x and y axes (Fig.) : Jr, + fro = mw, @ and NN, +Ny-mg-F= 0 or N,+N,= mg+F (2) Now choosing positive direction of @ as shown in the figure and using N.,= I, Bp we get mR? ,_ mR? We FR-(fr,+fr)R= B= ™ = @ [as for pure rolling w, = BR J. In addition to, Sry + fry sk (Ny + Ny) 4 1.257 1.258 129 Solving the Eqs., we get re 2kme yp pn 3kme @-3%) mx 23k kW, +Ny) wn ways et k k 3kmg)_ _2ke 7 pq U8 + Fax] ened 2-3k (a) Let us choose the positive direction of the rotation angle @, such that w,, and B, have identical signs (Fig.). Equation of motion, F, = mw,, and N,, = I. B, gives : F cosa - fr= mw,: frR-Fr= I,B,= ymR?B, In the absence of the slipping of the spool w,,= B,R From the three equations w,,= w,= P1SS2=C/R)I | where cos a> = (a) m(1+y¥) vy R (b) As static friction (fr) does not work on the spool, from the equation of the increment of mechanical energy A, = AT. Aue Smid ym? % tmasne : Fmt +y) 20, xe Fm(L+y2m (im 4 ") 2 Piles 2 fr “2m +9) ™g. Note|that at cos a= r/R, there is no rolling and for cos &<1r/R, Wz,<0, i.e. the spool will move towards negative x-axis and rotate in anticlockwise sense. For the cylinder from the equation N, = IB, about its stationary axis of rotation. 2Tr= Mp eS p- 2 Q) For the rotation of the lower cylinder from the equation N., = I. B, mr oy Dake 2Tr= - o, B= 7 =B Now for the translational motion of lower cylinder from the Eq. F,= mw,. mg -2T= mw, 2) As there is no slipping of threads on the cylinders : w= Bir+ Br= 2Br @ 130 1.259 1.260 Simultaneous solution of (1), (2) and (3) yields = OE T 10° Let us depict the forces acting on the pulley and weight A, and indicate positive direction for x and @ as shown in the figure. For the cylinder from the equation F,= m w and Nez= 1.8, we get Mg+T,-2T= Mw, @ Iw, and 27K +T,(2R)= 1B= = (2) | For the weight A from the equation F,= mw, mg -T,= mw, @ As there is no slipping of the threads on the pulleys. w= W,+2BR= w.+20,.=3w, (4) Simultaneous solutions of above four equations gives : 3(M+3m)g ‘A ae - (M+9m+—5! R (a) For the translational motion of the system (m,+ m,), from the equation : F,= mw, F= (m,+myw, or, w.= F/(m, +m) (1) Now for the rotational motion of cylinder from the equation : N. = I, By Pr Fre mp or pr= = (2) But We= w+ Br, So F_ .2F_ F(3m,+2m,) WK" Thm, my, (m, +m) 1 2 1 es a 2 @) (b) From the equation of increment of mechanical energy : AT= A, Here AT= T(t), 80, T()= Ag As force F is constant and is directed along x-axis the sought work done. Ag, = Fx (where x is the displacement of the point of application of the force F during time interval t ) 1,261 1.262 131 F'1?(3m,+2m,) 2m, (m, +m.) (using Eq. (3) Alternate : T(t) = Tyansiauon () + Trovangn () = F(Z = = T() 2 Ft Yimin Ft EG m+2m) (m+ 2-2 | mr 2m; (m,, m,) = pomem( Choosing the positive direction for x and @ as shown in Fig, let us we write the equation of motion for the sphere F,= mw,, and N,, =I, B, fray vy; frr= 2m? p (w, is the acceleration of the C.M. of sphere.) ~y For the plank from the Eq. F, = mw, (—+x F-f=m,, C;,) In addition, the condition for the absence of fr slipping of the sphere yields the kinematical from, ] relation between the accelerations : : e Ww, =w.t+Br d Simultaneous solution of the four equations yields : and p=2m (a) Let us depict the forces acting on the cylinder and their point of applications for the cylinder and indicate positive direction of x and @ as shown in the figure. From the equations for the plane motion of a solid F,= mw,, and N.. = 1,B,: kmg= mw, oF W= kg (1) 2 = kmg R= MR: or B,= -288 Q) Let the cylinder starts pure rolling at¢ = t after releasing on the horizontal floor at = 0. From the angular kinematical equation en (“+x or om ag-2 @) ®p From the equation of the linear kinematics, \ or v= O+kG fy (4) 132 1.263 1.264 But at the moment ¢= f, when pure rolling starts v,= wR so, kgty= (o0-2 feo} @R Thus b= 3k (b) As the cylinder pick, up speed till it starts rolling, the point of contact has a purely translatory movement equal to 5M. in the forward directions but there is also a backward movement of the point of contact of magnitude (09) - 5Ba) R. Because of slipping the net displacement is backwards. The total work done is then, Ag, = kang [». 8 - (ogo 2 ba] = kong [#8 : A BR - ooo] _ mate OR [oR ving SESE F ag) = The same result can also be obtained by the work-energy theorem, A, = AT. Let us write the equation of motion for the centre of the sphere at the moment of breaking-off: mv’/(R +1) = mg cos ®, where vis the velocity of the centre of the sphere at that moment, and @ is the corresponding angle (Fig.). The velocity v can be found from the energy conservation law : _ 5 a 5 10’, where J is the moment of inertia of the sphere relative to the axis passing through the sphere’s centre. ie. [= 2m, In addition, v= or; h= (R+r)(1-cos 6). From these four equations we obtain w= V10g(R+n)17P. Since the cylinder moves without sliding, the centre of the cylinder rotates about the point O, while passing through the common edge of the planes. In other words, the point O becomes the foot of the instantaneous axis of rotation of the cylinder. It at any instant during this motion the velocity of the C.M. is v, when the angle (shown in the figure) is B, we have my R = mgcosB-N, 1.265 133 where N is the normal reaction of the edge 2 NR - -_——_ 7 o, vw gReosp- 8B ty From the energy conservation law, 2 4 oe plo z to ze mg R (1 - cos B) 2 But y= ema? = 3 mk’, (from the parallel axis theorem) x Thus, Ya +S gR (1 - cos) 2 From (1) and (2) hm 2B (7e0sp-4 - ‘The angle B in this equation is clearly smaller than or equal to at so putting B = o we get ve 82 cosa 4) : aos where No is the corresponding reaction. Note that N 2 No. No jumping occurs during this turning if Ny > 0. Hence, vy must be less than Yaar VEE (1 008 4 4) Clearly the tendency of bouncing of the hoop will be maximum when the small body A, will be at the highest point of the hoop during its rolling motion. Let the velocity of C.M. of the hoop equal v at this position. The static friction does no work on the hoop, so from conservation of mechanical energy; E, = Ey 7 2 i 1 pete) _ of 2,1 Lire or, O+zmetzmR ( mgR zm (2vy tymvsomR (i) +mgR or, 3v= e-2gR aw From the equation F, = mw, for body A at final position 2 : 2 mer mo? R= m(R] R ) a 134 1.266 1.267 1.268 As the hoop has no acceleration in vertical direction, so for the hoop, N+N' = mg @) From Eqs. (2) and (3), Na 2mg-™ 4 R As the hoop does not bounce, Nz 0 6) So from Egs. (1), (4) and (5), Aiea 0 o 8gRzy Hence vos VEgR Since the lower part of the belt is in contact with the rigid floor, velocity of this part becomes zero. The crawler moves with velocity v, hence the velocity of upper part of the belt becomes 2v by the rolling condition and kinetic energy of upper part = ; 7 (2v)° = my’, which is also the sought kinetic energy, assuming that the length of the belt is much larger than the radius of the wheels. The sphere has two types of motion, one is the rotation about its own axis and the other is motion in a circle of radius R. Hence the sought kinetic energy T= hioittho @) where J, is the moment of inertia about its own axis, and J, is the moment of inertia about the vertical axis, passing through O, But, [y= 2m? and [y= Sma? + mR? (using parallel axis theorem,) Q) In addition to w= 5 and =z @) Using (2) and (3) in (1), we get T’ = % m? (: + ra For a point mass of mass dm, looked at from C rotating frame, the equation is dm" = F'+dmorr” +2dm(o" xB) where 7” = radius vector in the rotating frame with respect to rotation axis and ¥” = velocity in the same frame. The total centrifugal force is clearly Fey J dot” = mo? R, R, is the radius vector of the C.M. of the body with respect to rotation axis, also F< mv xo where we have used the definitions mR,= J, dmr” and mis’ = Y dmi™ 1.269 1.270 135 Consider a small element of length dx at a distance x from the point C, which is rotating in a circle of radius r= x sin @ Now, mass of the element = 7 a& ! So, centrifugal force acting on this element - 7 dco" xsin 0 and moment of this force about C, |av |= (7]rorssino-xeoso 2 MO sin 2x7 de and hence, total moment 12 2 mo. Jt pei N of By sin 20 Pde = 57 moo" l? sin 20, 0 Let us consider the system in a frame rotating with the rod. In this frame, the rod is at rest and experiences not only the gravitational force m Z’and the reaction force R, but also the centrifugal force F,,. In the considered frame, from the condition of equilibrium i.e. Ny, = 0 ia or, Nig = mg z sin ® where N,; is the moment of centrifugal force about O. To calculate N,, , let us consider an element of length de, situated at a distance x from the point O. This element is subjected to a horizontal pseudo force 7 dew" xsin ®. The moment of this pseudo force about the axis of rotation through the point O is aNy= (7) dx @? x sin 0.xcos 0 2 = TP sin cos @ x? dx 1 2 272 So ng= f 22 sin cos 07dr= 22 sind cos 0 Q) 0 It follows from Eqs. (1) and (2) that, cos 0 = (ch) or os xo""( 3745) () 2071 207 136 1271 1.272 1.273 When the cube is given an initial velocity on the table in some direction (as shown) it acquires an angular momentum about an axis on the table perpendicular to the initial velocity and (say) just below the C.G.. This angular momentum will disappear when the cube stops and this can only by due to a torque. Frictional forces cannot do this by themselves because they act in the plain containing the axis. But if the force of normal reaction act eccentrically (as shown), their torque can bring about the vanishing of the angular momentum. We can calculate the distance Ax between the point of application of the normal reaction and the C.G. of the cube as follows. Take the moment about Jnitial velocity — Axis. to the CG. of all the forces. This must vanish because Initial iar vane the cube does not turn or tumble on the table. angular ake (ation, Then if the force of friction is fr momentum a fr ra Nk& But N= mg and fr= long, so Ax= ka/2 In the process of motion of the given system the kinetic energy and the angular momentum relative to rotation axis do not vary. Hence, it follows that 1Me 5 1 3 7 Mie 7 ZOO = FM OP +7) 4 Tw (@ is the final angular velocity of the rod) Ml Ml 2 and Fo Ot ml? w From these equations we obtain o= o/ (1 + ar) v= wgt/ Vi+3m/M Due to hitting of the ball, the angular impulse received by the rod about the C.M. is equal top i If is the angular velocity acquired by the rod, we have ml? i 6 et o or o= = a In the frame of C.M., the rod is rotating about an axis passing through its mid point with the angular velocity @. Hence the force exerted by one half on the other = mass of one half x acceleration of C.M. of that part, in the frame of C.M. m( zt ot | 9p? | - 3073) ce 1274 1.275 137 (a) In the process of motion of the given system the kinetic energy and the angular momentum relative to rotation axis do not Ts Hence it follows that 1 av? = dL my?42 ut w? 2 2 2\ 3 1 Ms and moze D Ze From these equations we obtain vm (28=4)y and w = ——# —__ 3m+ 4m |" 1(1+4m/3M) > aa > 3m - 4M \—» As vt tv, so in vector form v= 3maam |” (b) Obviously the sought force provides the centripetal acceleration to ‘the C.M. of the rod and is F,= mWeq et 2° 1(1+4M/3m) (a) About the axis of rotation of the rod, the angular momentum of the system is conserved. Thus if the velocity of the flying bullet is v. 2 mvl = (m2+43-) 3 Seta ed M 1 a nM as m << q@) most Now from the conservation of mechanical energy ofthe system (rod with bullet) in the uniform field of gravity a(mie4s i? Jo? = (Mamet (1 cose.) Q) [because C.M. of rod raises by the height 4 1-cose) ] Solving (1) and (2), we get M) 4/2 9 sin : (3) 38! sing and @ (b) Sought Ap = [con ea(o3)]-m where ol is the velocity of the bullet and « 5 equals the velocity of C.M. of the rod after the impact. Putting the value of v and w we get ae ! sin & Ap~ 5mv=M & sin This is caused by the reaction at the hinge on the upper end. 138 1.276 1.277 (©) Let the rod starts swinging with angular velocity w’, in this case. Then, like part (a) ( mt ‘| 3 myx = ay mx On ee eee Final momentum is 1 M M 3 x p= mo! +fyc! Maye Mot tm anv o I 2 2 I So, Ap= p,-vm mv (35-1) on 2 This vanishes for xeGl (a) As force F on the body is radial so its angular momentum about the axis becomes zero and the angular momentum of the system about the given axis is conserved. Thus MR? Ro oon 1428 2 Or (b) From the equation of the increment of the mechanical energy of the system : AT= A, 2 ©) +m oR? = 1MR? 2 1(MR? 2322 2\ 2 2) 52 +mR Jed Ae Putting the value of « from part (a) and solving we get BSE (1m) M oad 2 (a) Let z be the rotation axis of disc and @ be its rotation angle in accordance with right-hand screw rule (Fig.). (p and g’ are to be measured in the same sense algebraically.) As M, of the system (disc + man) is conserved and M, gia) ™ 0, We have at any instant, mR? do do’) 2, (22 00 em dt aka d mi —_|q . P| my + (m2) |°* On integrating Jere Nastia) mt ’ or, en m\? a) m+ S This gives the total angle of rotation of the disc. 1.278 1.279 139 (b) From Eq. (1) ag m \do'_ _(_™_)\v¥(r) a dt R m+ 2 m+ 2 Differentiating with respect to time 3 ao m,_\1dv'(t) dt? m, R dt m+ 2 Thus the sought force moment from the Eq. N,= 1B, _ R? q? R? eas er ae m,_\idv(t) 0, the corresponding vector @ coincides with the poitive direction to the z axis, and vice versa. As both discs rotates about the same vertical axis z, thus in vector form. B= 8, +1,0,/(h+h) However, the problem makes sense only if @, tt @, or @, t) @, (0) From the equation of increment of mechanical energy of a system: Aj, = AT. 1 1 io = 7Uith)o?-shogtzhoz Using Eq. (1) f, 1 L, 2 2 Age FU Fhy (Ou Ors) For the closed system (disc + rod), the angular momentum is conserved about any axis. Thus from the conservation of angular momentum of the system about the rotation axis of rod passing through its C.M. gives : L 1 yml? mas my 5+ (1)

Anda mungkin juga menyukai